Download as pdf or txt
Download as pdf or txt
You are on page 1of 58

UPSC CSE EXAM

9 Year TOPIC - Wise PDF


Prelims PYQs
Content
(SUBJECT-WISE & TOPIC-WISE)

Modern History
[Part of our Prelims PYQ Content]
PRELIMS PYQs Content- (Modern History)

INDEX

CHAPTER 1: INDIA ON THE EVE OF BRITISH CONQUEST------------------------------------ 4


CHAPTER 2: IMPORTANT PERSONALITIES -------------------------------------------------------- 6
CHAPTER 3: CONSTITUTIONAL REFORMS DURING BRITISH INDIA ------------------ 23
CHAPTER 4: BRITISH EXPANSION AND CONSOLIDATION IN INDIA ------------------- 29
CHAPTER 5: POPULAR UPRISINGS UPTO 1857 -------------------------------------------------- 30
CHAPTER 6: POPULAR UPRISINGS AFTER 1857 (1857-1947) -------------------------------- 31
CHAPTER 7: SOCIO-RELIGIOUS REFORM MOVEMENTS ----------------------------------- 33
CHAPTER 8: RISE OF MILITANT NATIONALISM ----------------------------------------------- 34
CHAPTER 9: FIRST PHASE OF REVOLUTIONARY ACTIVITIES (1907-17) -------------- 36
CHAPTER 10: MAHATMA GANDHI: EMERGENCE AND IDEOLOGY -------------------- 37
CHAPTER 11: NATIONALIST RESPONSE IN WAKE OF SECOND WORLD WAR ---- 39
CHAPTER 12: CDM AND ROUND TABLE CONFERENCES ----------------------------------- 41
CHAPTER 13: QUIT INDIA MOVEMENT ------------------------------------------------------------ 42
CHAPTER 14: IMPORTANT CONGRESS SESSIONS --------------------------------------------- 43
CHAPTER 15: ORGANIZATIONS----------------------------------------------------------------------- 45
CHAPTER 16: EVOLUTION OF CIVIL SERVICES IN INDIA ---------------------------------- 48
CHAPTER 17: DEVELOPMENT OF EDUCATION ------------------------------------------------- 49
CHAPTER 18: ECONOMIC IMPACT OF BRITISH RULE IN INDIA ------------------------- 51
CHAPTER 19: MISCELLANEOUS ---------------------------------------------------------------------- 53

THESE PDFs are component of our PRELIMS PYQ Content

Content made from Every word of Previous Year Paper | www.sunyaias.com | 8279688595
Page. 2
PRELIMS PYQs Content- (Modern History)
CHAPTER 1: INDIA ON THE EVE OF BRITISH CONQUEST
[Portuguese, Dutch, English, Danes & French]
PRELIMS PYQ CONTENT – UPSC CSE EXAM – Topic-wise Segregated

1. Factories of English East India Company: 4. In the first quarter of the seventeenth
By 1623, English East India Company had century, in which of the following
established factories at Surat, Broach, was/were the factory/factories of the
Ahmedabad, Baroda, Agra, Masulipatam and English East India Company located?
Armagaon near Pulicat + The English opened 1. Broach
their first permanent British factory at Surat 2. Chicacole
in 1613. [UPSC 2021] 3. Trichinopoly
2. Factories of Portuguese: Portuguese Select the correct answer using the code
established their first factory in India at given below:
Calicut in 1500 AD. This factory was (a) 1 only
abandoned in 1525 AD due to the opposition (b) 1 and 2
of Zamorin of Calicut. (c) 3 only
3. Factories of Dutch: In India, they (d) 2 and 3
established the first factory in
Masulipattanam in 1605, followed by Pulicat
in 1610, Surat in 1616, Bimilipatam in 1641
and Chinsura in 1653. In Bengal they
established a factory in Pipli, but it was
abandoned for Balasore later.
4. Factories of Danish: In 1616 AD, the
Danish East India company established their
first factory at Tranquebar in Tamilnadu
(1620) and Serampur in Bengal (1676).
5. Francis Xavier: He led an extensive mission 3. Consider the following statements:
into Asia, mainly in the Portuguese Empire of 1. St. Francis Xavier was one of the
the time and was influential in evangelization founding members of the Jesuit
work, most notably in India. He was the first Order.
Christian missionary to venture into Japan, 2. St. Francis Xavier died in Goa and a
Borneo, the Maluku Islands, and other areas. church is dedicated to him there.
Known as the ―Apostle of the Indies‖ and 3. The feast of St. Francis Xavier is
―Apostle of Japan‖, he is considered to be one celebrated in Goa each year.
of the greatest missionaries since Paul the Which of the statements given above are
Apostle + Saint Francis Xavier, was a Catholic correct?
missionary and saint who was a co-founder of (a) 1 and 2 only
the Society of Jesus. He was born in Javier and (b) 2 and 3 only
was a companion of Ignatius of Loyola and one (c) 1 and 3 only
of the first seven Jesuits + Francis died in the (d) 1, 2 and 3
island of Sancian, off the Chinese coast + Feast
of St. Francis Xavier is observed on Dec 3rd
each year in Goa. [UPSC 2021]
6. Gajapati rulers 1. With reference to Indian history, consider
 Kapilendra, the minister of Bhanudeva IV the following statements:
(1414-1435), usurped the throne in 1435 1. Dutch established their factories/
and laid the foundation of the Gajapati rule warehouses on the east coast on lands
in Odisha. granted to them by Gajapati rulers.

Content made from Every word of Previous Year Paper | www.sunyaias.com | 8279688595
Page. 3
PRELIMS PYQs Content- (Modern History)
 After Pratapa Rudra‘s death (1540), his 2. Alfonso de Albuquerque captured
successors could hardly hold the empire Goa from the Bijapur Sultanate.
intact, and the end of Suryavamsi 3. The English East India. Company
(Gajapati) dynasty came soon after established a factory at Madras on a
(1542), whereas the Dutch East India plot of land leased from a
Company was formed in 1602 through a representative of the Vijayanagara
charter. empire.
 Albuquerque acquired Goa from the Which of the statements given above are
Sultan of Bijapur in 1510 with ease; the correct?
principal port of the Sultan of Bijapur (a) 1 and 2 only
became ―the first bit of Indian territory to (b) 2 and 3 only
be under the Europeans since the time of (c) 1 and 3 only
Alexander the Great‖. [UPSC 2022] (d) 1, 2 and 3
7. Staple Commodities of Export by EEIC: 2. The staple commodities of export by the
The East India Company was originally English East India Company from Bengal
formed in Britain for pursuing trade with the in the middle of the 18th century were
East Indies in Southeast Asia. Cotton, raw (a) Raw cotton, oil-seeds, and opium
silk, saltpetre, opium were the major (b) Sugar, salt, zinc, and lead
commodities exported from Bengal by the (c) Copper, silver, gold, spices, and tea
English East India Company. [UPSC 2018] (d) Cotton, silk, saltpeter, and opium

*******

Content made from Every word of Previous Year Paper | www.sunyaias.com | 8279688595
Page. 4
PRELIMS PYQs Content- (Modern History)
CHAPTER 2: IMPORTANT PERSONALITIES
PRELIMS PYQ CONTENT – UPSC CSE EXAM – Topic-wise Segregated

1. Rabindranath Tagore: Rabindranath 9. With reference to Madanapalle of Andhra


Tagore translated "Jana Gana Mana" from Pradesh, which one of the following
Bengali to English as ‗Morning song of statements is correct?
India‘ and also set it to music in (a) Pingali Venkayya designed the
Madanapalle. National Anthem was written tricolour Indian National Flag here.
by Rabindranath Tagore and was first sung (b) Pattabhi Sitaramaiah led the Quit
on December 27, 1911 at the Calcutta India Movement of Andhra region
session of the Indian National Congress. The from here.
Hindi version of National Anthem was (c) Rabindranath Tagore translated
adopted by the Constituent Assembly as the the National Anthem from Bengali
National Anthem of India on January 24, to English here.
1950. [UPSC 2021] (d) Madame Blavatsky and Colonel
 The parent song, ‗Bharoto Bhagyo Olcott set up headquarters of
Bidhata‘ is a Brahmo hymn which has five Theosophical Society first here.
verses and only the first verse has been
adopted as National Anthem.
 In 1912, the song was published under the
title Bharat Bidhata in the Tatwabodhini
Patrika, which was the official publication
of the Brahmo Samaj and of which Tagore
was the Editor.
 Playing time of full version of National
Anthem is 52 seconds and A short version
consisting of first and last lines of the
stanza must be played in about 20 seconds.
2. Pingali Venkayya: The Indian tricolour was
designed by Pingali Venkayya, who was a
freedom fighter and was a follower of
Mahatma Gandhi. He presented it to
Mahatma Gandhi in 1921 during the session
of the All India Congress Committee in
Vijaywada. The flag, at that time, was made
of Green and Saffron colour, representing
Muslim and Hindu communities of India. but
it later evolved with a spinning wheel at the
centre and a third colour-white. The flag was
officially adopted by the Indian National
Congress in 1931. [UPSC 2021]
 Pingali Venkayya, Born on August 2,
1876 in Krishna district of Andhra
Pradesh, Venkayya served as a soldier in
the British Army in South Africa during
the Anglo Boer war in Africa.
3. The Quit India Movement in Andhra
 In Andhra the Provincial Congress
Committee had issued a circular

Content made from Every word of Previous Year Paper | www.sunyaias.com | 8279688595
Page. 5
PRELIMS PYQs Content- (Modern History)
popularly known as ‗Kurnool Circular‘
as the police ceased the copy when they
ride ‗Kurnool Congress Office. This was
drafted by Kala Venkat Rao, on 29th July
1942 and was sent for approval of
Congress Working Committee through
Dr. Pattabhi Sitaramaiah, member of
working committee.
 The ‗Kurnool Circular‘ envisaged a
programme of defying prohibitory
orders, lawyers to give up practice,
students to leave colleges, picketing salt
and foreign trade and industry, cutting of
communications, cutting of toddy
yielding trees, travelling without tickets,
pulling chains to stop trains and blow up
bridges to disrupt communications and
retard the movement of Army personnel
 Some of the prominent leaders who were
taken as detenus during this period in
Andhra were Pattabhi Sitaramaiah,
A.Kaleshwer Rao, T.Prakasham,
Neelam Sanjeeva Reddy, Maganti
Bapineedu and several others.
4. Pattabhi Sitaramaiah: Indian Nationalism
(1913), The Redistribution of Indian
Provinces on a Linguistic Basis (1916) and
Non Cooperation (1921) were famous books
penned by Pattabhi Sitaramayya. He was
also the founder of the Andhra Bank in 1923
in Masulipatnam. He also founded ‗Krishna
Cooperative Central Bank‘ in 1915, Andhra
Insurance Company in 1925 and Bharat
Laxmi Bank Ltd. in 1929 etc. [UPSC 2021]
5. Madame Blavatsky and Olcott: e
Theosophical Society was founded by
Madame H. P. Blavatsky and Colonel Olcott
in New York in 1875. In 1882, headquarters
were established in Adyar, near Madras (now
Chennai) in India. [UPSC 2021]
6. MK Gandhi: Gandhiji was a prolific writer 10. Who among the following is associated
and he has written many articles throughout with ‗Songs from Prison‘, a translation of
his life. He edited several newspapers ancient Indian religious lyrics in English?
including Harijan in Gujarati, Indian opinion (a) Bal Gangadhar Tilak
in South Africa, and Young India in English (b) Jawaharlal Nehru
+ He wrote several books including his (c) Mohandas Karamchand Gandhi
autobiography ―The Story of My (d) Sarojini Naidu
Experiments with Truth‖ + Mahatma Gandhi
was associated with ―Songs from Prison‖, a 11. Who among the following was associated

Content made from Every word of Previous Year Paper | www.sunyaias.com | 8279688595
Page. 6
PRELIMS PYQs Content- (Modern History)
translation of ancient Indian religious lyrics as Secretary with Hindu Female School
in English in 1934. [UPSC 2021] which later came to be known as Bethune
7. Bal Gangadhar Tilak: He was an Indian Female School?
nationalist, teacher and an independence (a) Annie Besant
activist conferred with the title of Lokmanya (b) Debandranath Tagore
+ He founded the Deccan Education Society (c) Ishwar Chandra Vidyasagar
in 1884 along with his associate Gopal (d) Sarojini Naidu
Ganesh Agarkar and others to create a new
system of education that taught young
Indians nationalist ideas through an emphasis
on Indian culture + Tilak helped found the
All India Home Rule League in 1916–18
with G. S. Khaparde and Annie Besant +
Tilak‘s Home Rule League worked in
Maharashtra, Central Provinces and
Karnataka and Berar region. Besant‘s
League was active in the rest of India +
Newspapers: Kesari (Marathi) and Mahratta
(English) + Books: Shrimadh Bhagvad Gita
Rahasya and Arctic Home of the Vedas.
8. Sarojini Naidu: Sarojini Naidu was an
Indian independence activist, poet and
politician + ‗The Nightingale of India‘ or
'Bharat Kokila', a name given by Mahatama
Gandhi + She wrote the play "Maher
Muneer"; she became the second woman
president of the Indian National Congress. +
She was one of the major figures to have led
the Civil Disobedience Movement and the
Quit India Movement + She was
instrumental in establishing the Women‘s
India Association, founded by Annie Besant
in 1917 + She was appointed the President of
the Indian National Congress in 1925 and
later became the Governor of the United
Provinces in 1947, becoming the first woman
to hold the office of Governor in the
Dominion of India.
9. Jawaharlal Nehru: He was a nationalist
leader, social democrat, author, and humanist
+ Ferdinand T. Brooks made impression
him + He would tell a British judge -"I
returned to India as much prejudiced in
favour of England & English as it was
possible for an Indian to be"+ He was a
prolific writer and authored books such as
‗The Discovery of India‘ ,‗Glimpse, The
Unity of India, A Bunch of Old Letters, 'Will
& Testament', The Autobiography + First

Content made from Every word of Previous Year Paper | www.sunyaias.com | 8279688595
Page. 7
PRELIMS PYQs Content- (Modern History)
attracted by Gokhale's 'Servants of India
Society.'(Not joined) + Lucknow
session(1916)- Nehru met Gandhi for first
time + 1916- Became joint secretary of
provincial Home Rule League + Jallianwala
Bagh tragedy brought Nehru into close
contact with Gandhi + 1927, appointed as
Congress representative at Congress of
Oppressed Nationalities, Brussels + From
prison, Nehru's letters to Indira- Glimpses of
World History + Nehru was elected
President of the Lahore Session of INC
(1929), where complete independence for the
country was adopted as the goal + He was
elected President of the Lucknow Session in
1936 again, here he urged Congress for
socialism as its goal + Became President of
All-India States People's Conference in
1939 + 1946, an Interim Cabinet, headed by
Nehru + Since August 15, 1947 he was PM
till his death on May 27, 1964 (17years) +
His critics describe him as the 'Harnlet of
Indian Polities'. + Nehru Stressed need for
atomic energy + He asked Dr D.S. Kothari
to study nuclear, thermonuclear & other
weapons of mass destruction + Close
relationship with Einstein, Bertrand Russell,
Henry Dale, Alexander Fleming, A.V. Hill,
Frederic Joliot-Curie, Niels Bohr, J.D.
Bernal, J.B.S. Haldane and P.M.S. Blackett +
Also called as ‗Child of the Indian
renaissance‘
10. Debendranath Tagore: Debendranath was
the eldest son of Prince Dwarkanath Tagore,
who was often referred to as the "Indian
Croesus" and was born in Jorasanko, in
Calcutta, West Bengal + In October 1839, he
along with his friends started the
Tattwaranjini Sabha which was later
renamed to Tattwabodhini Sabha. Its object
was the dissemination of the knowledge of
the Upanishads and promotion of religious
enquiry.
11. Ishwar Chandra Vidyasagar: In his book,
"Barno-Porichoy" (Introduction to the letter),
Vidyasagar refined the Bengali language and
made it accessible to the common strata of
the society + He proposed and pushed for the
Widow Remarriage Act of 1856 in India and

Content made from Every word of Previous Year Paper | www.sunyaias.com | 8279688595
Page. 8
PRELIMS PYQs Content- (Modern History)
raised concern for the abolition of child
marriage and polygamy + He also opened the
doors of the colleges and other educational
institutions to lower caste students, which
was earlier reserved only for the Brahmins +
Pandit Iswar Chandra Vidyasagar was the
first Secretary of Bethune Female School.
12. Annie Besant: She founded the All India
Home Rule League in 1916. This was India's
first faction to demand complete
independence + She was one of the founders
of the Banaras Hindu University + She also
founded the Central Hindu School to
promote education + By 1918, she had
founded Madras Parliament, Madanapalle
College (now in Andhra Pradesh), Adyar
Arts League, Bombay Home Rule League,
Women's Indian Association at Adyar and
the Women's Indian Association at Adyar.
13. Gandhi 6. With reference to the British colonial rule
 Gandhi was sent to school at the age of in India, consider following statements:
seven. He married Kasturba at the age of 1. Mahatma Gandhi was instrumental in
12. Gandhi passed his matriculation the abolition of the system of
examination in 1888 and was sent to ‗indentured labour‘
Bhavnagar for higher studies but could 2. In Lord Chelmsford‘s ‗War
not adjust there. It was at this time that Conference‘, Mahatma Gandhi did not
Mavji Dave, a family friend, suggested support the resolution on recruiting
that Gandhi be sent to England. Indians for World War.
 In England he made friends with 3. Consequent upon the breaking of Salt
Theosophists and was introduced to Law by Indian people, the Indian
Madame Blavatsky and Mrs. Annie National Congress was declared
Besant. illegal by the colonial rulers.
 After his contact with men of different Which of the statements given above are
faiths and studying various religious correct?
books, Gandhi said that he had crossed (a) 1 and 2 only
the 'Sahara of Atheism'. He sailed back (b) 1 and 3 only
to India in 1891. (c) 2 and 3 only
 He initially went to South Africa for one (d) 1, 2 and 3
year to attend to a legal matter of Dada
Abdullah Jhaveri, but stayed on with a
few breaks for about 21 years [1893-
1914).
 In the Boer War, though Gandhi's real
sympathy was with the Boers, he decided
to lend support to the British as a citizen
of the British Empire. He founded an
Ambulance Corps of around 1100
volunteers in support of the British in the
Anglo-Boer War of 1899-1902.

Content made from Every word of Previous Year Paper | www.sunyaias.com | 8279688595
Page. 9
PRELIMS PYQs Content- (Modern History)
 The 1901 Calcutta Session was the first
time Mahatma Gandhi appeared on the
Congress platform. Then a lawyer based
in South Africa, Gandhi urged the
Congress to support the struggle against
racial discrimination and exploitation in
the country. Gandhi returned back to S.
Africa in 1902.
 In South Africa, Gandhi first tried out
satyagraha methods to fight against the
racialism prevalent there. He
experimented with the idea of
community living at the Phoenix Farm.
Later, in 1910 he established Tolstoy
Farm in Transvaal.
14. Indentured labour [UPSC 2019]
 It was a system of bonded labour
introduced after abolition of slavery as
newly free men refused to work for low
wages on sugar and tea plantations in
British colonies of West Indies, Fiji,
Mauritius and Ceylon.
 These people were termed ‗girmitiyas‘.
 In South Africa, Gandhi in opposition to
compulsory registration of Indian
population under Asiatic Law
Amendment Act organised non-violent
protest -> led to passage of Indian Relief
Act (1914) which eliminated extra tax on
Indian citizens who had not renewed
their indentures and recognised validity
of Indian marriages.
15. Indian National congress was declared illegal
on two instances once during Salt satyagraha
and during Quit India Movement.
16. During World war I in the spring of 1917
Germany had inflicted crushing defeats on
both the British and French troops in France.
The Viceroy of India Lord Chelmsford,
invited various Indian leaders to attend a
War conference. Gandhi was also invited. He
accepted the invitation and went to Delhi
Gandhi was not happy that leaders like Tilak
or the Ali brothers had not been invited to
the conference, so he felt unwilling to attend.
After meeting the Viceroy, however, he
attended the conference.
 The Viceroy was very keen that Gandhi
should support the resolution on

Content made from Every word of Previous Year Paper | www.sunyaias.com | 8279688595
Page. 10
PRELIMS PYQs Content- (Modern History)
recruiting Indians. Gandhi spoke only
one sentence: 'With a full sense of my
responsibility I beg to support the
resolution.'
 He issued leaflets asking people to enlist
in the forces.
17. Prem Kumar Sehgal, Shah Nawaz Khan: 12. In the context of Colonial India, Shah
In 1939, Subash Chandra Bose resigned from Nawaz Khan, Prem Kumar Sehgal and
the congress and organised Forward Bloc for Gurbaksh Singh Dhillon are remembered
a more vigorous national struggle. In 1943 as
he reached Japan and organise an armed (a) leaders of Swadeshi and Boycott
struggle against British rule with Japanese Movement
help. In 1942 Capitan Mohan Singh formed (b) members of the Interim Government
INA comprising the captured Indian soldiers in 1946
along with Indian Independence league of (c) members of the Drafting Committee
Rash Bihari Bose. Rash Behari Bose handed in the Constituent Assembly
over control of the organization to Subhas (d) officers of the Indian National
Chandra Bose. In 1944, the INA tried to Army
invade India through Imphal and Kohima but
the campaign failed. The INA members were
imprisoned and tried. Officers of INA:
Prem Kumar Sehgal, Shah Nawaz Khan
and Gurubaksh Singh Dhillon were
officers of Indian National Army whose trial
were held in 1945 at the Red Fort in Delhi.
This was the first trial of INA Prisoners of
wars. [UPSC 2021]
18. Sakharam Ganesh Deuskar (1869-1912): 8. With reference to the book ―Desher Katha‖
He was a close associate of Sri Aurobindo; written by Sakharam Ganesh Deuskar
published a book entitled ‗Desher Katha‘ during the freedom struggle, consider the
(Story of the Nation) in 1904 -> described following statements :
about British commercial and industrial 1. It warned against the Colonial State‘s
exploitation of India; it was banned by hypnotic conquest of the mind.
government of Bengal in 1910 + The book 2. It inspired the performance of
has captured the mind of young Bengal and swadeshi street plays and folk songs.
assisted in preparation of the Swadeshi 3. The use of ‗desh‘ by Deuskar was in
Movement + Deuskar uses ‗desh‘ in the specific context of region of Bengal.
context of the whole country + He Which of given statements are correct?
popularized the ideas of Naoroji and Ranade (a) 1 and 2 only
and promoted swadeshi in a popular idiom + (b) 2 and 3 only
He was the first person to bring the name of (c) 1 and 3 only
‗Swaraj‘. [UPSC 2020] (d) 1, 2 and 3
19. Sir Tej Bahadur Sapru: Tej Bahadur Sapru 7. With reference to Indian National
was a prominent member of Liberal Party of Movement, consider the following pairs:
India and the President of All India Liberal Person Position Held
Federation [UPSC 2019] + He believed that 1. Sir Tej President, All India
dialogue with the British authorities was the Bahadur Sapru Liberal Federation
only manner in which political rights and 2. K. C. Neogy Member, Constituent
freedom could be achieved for the country + He Assembly

Content made from Every word of Previous Year Paper | www.sunyaias.com | 8279688595
Page. 11
PRELIMS PYQs Content- (Modern History)
collaborated with the Congress on various 3. P. C. Joshi General Secretary,
movements like the non-cooperation Communist Party of
movement, salt satyagraha and the Quit India India
Movement + He mediated between Mahatma Which of the pairs given above is/are
Gandhi and Lord Irwin and facilitated correctly matched?
the Gandhi-Irwin pact. He also intervened (a) 1 only
between Gandhi and Dr Ambedkar over the (b) 1 and 2 only
issue of separate electorates for the Depressed (c) 3 only
Classes and helped bring about the Poona Pact (d) 1, 2 and 3
+ He defended rebel soldiers of Indian National
Army in Meerut conspiracy case. He
established Indian Council of World Affairs in
1943 and served as its president till 1949.
20. KC Neogy: He was a member of INC and
was elected as a member of the Central
Legislative Assembly representing Bengal +
He attended the Round Table
Conferences representing the Orissa States +
He was elected as a member of
the Constituent Assembly of India and after
independence became a member of the First
Cabinet of Independent
India under Jawaharlal Nehru as the Minister
of Relief and Rehabilitation and later as
Minister for Commerce. [UPSC 2019]
21. P.C Joshi: P.C. Joshi joined the Communist
Party formally in 1929. Immediately after in
March 1929, along with 31 other
Communists and trade union workers, he
was arrested and tried in the famous Meerut
Conspiracy Case. [UPSC 2019]
22. Bankim Chandra Chattopadhyay: He 6. Which among following events happened
composed the song Vande Mataram in earliest?
Sanskrit, which was a source of inspiration (a) Swami Dayanand established Arya
to the people in their freedom struggle. Samaj.
Founded a monthly literary magazine, (b) Dinabandhu Mitra wrote
Bangadarshan, in 1872. His famous novels Neeldarpan.
include Kapalkundala (1866), Debi (c) Bankim Chandra Chattopadhyay
Choudhurani, Bishabriksha (The Poison wrote Anandmath.
Tree), Chandrasekhar (1877), Rajmohan‘s (d) Satyendranath Tagore became the first
wife and Krishnakanter Will. [UPSC 2018] Indian to succeed in the Indian Civil
23. Satyendranath tagore: Satyendranath Tagore Services Examination.
was the first Indian to qualify the Civil Services
Examination in 1864. He was India's first Indian
Administrative Service (IAS) officer. [UPSC
2018]
24. Arya Samaj
 Swami Dayananda Saraswati (1824-83)
founded the Arya Samaj in 1875. [UPSC

Content made from Every word of Previous Year Paper | www.sunyaias.com | 8279688595
Page. 12
PRELIMS PYQs Content- (Modern History)
2018]
 Believes in the infallibility of the Vedas
and regards them as the ultimate source
of all truth and knowledge.
 Opposes God's idolatry and reincarnation
idea, but supports the notion of 'Karma'
and soul transmigration.
 Rejects Brahmanical domination over
Hindu spiritual and social life. Brahmins'
claim to be conduits between man and God
is condemned.
 Supported the Four Varna
System, however, it should be based on
merit rather than birth.
 Advocated for women's equality in
society. There is no place for any type of
gender discrimination against women.
 Supported widow remarriage and female
education while opposing polygamy,
child marriage, Sati, and other practices.
25. Neel darpan: The Indigo revolt (1859)
directed against European planters exploited
the local peasants by forcing them to take
advances and sign fraudulent contracts
according to which the peasants were
compelled to grow Indigo, rather than the
more profitable rice. Din Bandhu Mitra
wrote a play ‗Neel Darpan‘ to portray the
oppression of indigo farmers. [UPSC 2018]
26. Lala Lajpat Rai 5. He wrote biographies of Mazzini,
 Also called Punjab Kesari and Lion of Garibaldi, Shivaji and Shrikrishna; stayed
Punjab. in America for some time; and was also
 He was influenced by Swami Dayananda elected to the Central Assembly. He was
Saraswati and joined the Arya Samaj in (a) Aurobindo Ghosh
Lahore. (b) Bipin Chandra Pal
 He believed that the ideals in Hinduism (c) Lala Lajpat Rai
combined with nationalism will lead to (d) Motilal Nehru
the establishment of a secular state.
 Along with Bipin Chandra Pal and Bal
Gangadhar Tilak, he formed the Lal-Bal-
Pal trio of extremist leaders.
 He founded the Home Rule League of
America in 1917 in New York.
 He was also elected President of the All
India Trade Union Congress.
 He supported the non-cooperation
movement of Gandhi at the Nagpur
session of the Congress in 1920.

Content made from Every word of Previous Year Paper | www.sunyaias.com | 8279688595
Page. 13
PRELIMS PYQs Content- (Modern History)
 He was elected deputy leader of Central
Legislative Assembly in 1926.
 In 1928, he moved a resolution in the
assembly refusing cooperation with the
Simon Commission since the
Commission had no Indian members.
 He founded Hindu Relief movement in
1897 to provide help to the famine -
stricken people and thus preventing them
falling into the clutches of the
missionaries.
 He founded the Servants of People Society
in 1921.
 In 1885, Rai established the Dayanand
Anglo-Vedic School in Lahore and
remained a committed educationist
throughout his life.
 Literary works: ‗The Arya Samaj‘,
‗Young India‘, ‗England‘s Debt to
India‘, ‗Evolution of Japan‘, ‗India‘s
Will to Freedom‘, ‗Message of the
Bhagwad Gita‘, ‗Political Future of
India‘, ‗Problem of National Education
in India‘, ‗The Depressed Glasses‘, and
the travelogue ‗United States of
America‘. [UPSC 2018]
 In 1896, he wrote a series of books in
Urdu, which he named the ‗Great Men of
the World‘. The first great man he chose
was Mazzini, and then Garibaldi, along
with Sivaji, Dayananda and Sri Krishna.
27. Bipin Chandra Pal
 He started popularising the idea of
‗swaraj‘ to Indians.
 He edited the journals ‗The Democrat‘
and ‗The Independent‘ and started many
newspapers and magazines such as
‗Paridarsak‘, ‗Bande Mataram‘, ‗New
India‘ and ‗Swaraj‘.
 Books: ‗Indian Nationalism‘,
‗Swaraj and the Present Situation,
‗Nationality and Empire‘, ‗The Basis of
Social Reform‘, ‗The New Spirit and
Studies in Hinduism‘, and ‗The Soul of
India‘.
 He joined the Congress Party in 1886. He
also participated in the Swadeshi
movement and the non-cooperation

Content made from Every word of Previous Year Paper | www.sunyaias.com | 8279688595
Page. 14
PRELIMS PYQs Content- (Modern History)
movement. He agitated against the
partition of Bengal of 1905.
 Pal was a fierce critic of Mahatma
Gandhi and his pacifist ways.
 Aurobindo Ghosh called Pal ‗one of the
mightiest prophets of nationalism‘.
28. Aurobindo Ghosh
 From 1902 to 1910 he partook in the
struggle to free India from the British.
 He was much influenced by the
American Revolution, revolts in Italy and
the French revolts against England.
 He founded a community of spiritual
seekers in Pondicherry, which later took
shape as the Sri Aurobindo Ashram in
1926.
 He helped establish the Anushilan
Samiti of Calcutta in 1902.
 He was edited newspapers such as Bande
Mataram and published magazine Arya;
greatest literary achievement
was ‗Savitri‘, an epic poem with about
24000 lines.
 In May 1908, Aurobindo was arrested in
connection to Alipore Conspiracy Case.
 Sri Aurobindo Ghosh was considered
as a prophet of Indian nationalism. It
was based on Vedanta philosophy which
saw unity and oneness in man and God.
 Literary works: Bande Mataram, Bases
of Yoga, Bhagavad Gita and Its Message,
The Future Evolution of Man, Rebirth
and Karma, Savitri: A Legend and a
Symbol and Hour of God.
29. Motilal Nehru
 He served as Indian National Congress
(INC) president twice, for the first time
in 1919 (Amritsar) and then in 1928
(Calcutta).
 He was president of the Allahabad
branch of the Home Rule League started
by Annie Besant.
 When Gandhi cancelled NCM, he
formed the Swarajya Party along with
CR Das.
 He became a member of the Central
Legislative Assembly in 1923 serving as
the Leader of the Opposition.

Content made from Every word of Previous Year Paper | www.sunyaias.com | 8279688595
Page. 15
PRELIMS PYQs Content- (Modern History)
 Nehru Report (1928)
Recommendations
o India should be granted Dominion
Status with a Parliamentary form of
government and a bi-cameral
legislature consisting of a Senate and
a House of Representatives.
o In India, a federal form of
government should be established,
with the Centre retaining residuary
powers.
o No separate electorate for minorities
and a joint electorate should be
established.
o No reserved seats for Punjabi and
Bengali communities. However,
Muslim seats may be reserved in
provinces with a Muslim population
of at least 10%.
o The judiciary must be separate from
the executive.
30. Radhakant Deb (1783-1867) 2. Consider the following pairs:
 He was a Bengali reformer and cultural 1. Radhakanta Deb — First President of
nationalist who dedicated his life to the the British Indian Association
preservation of orthodox Hinduism. 2. Gazulu Lakshminarasu Chetty —
 He was the first president of British Founder of the Madras Mahajana
Indian Association. [UPSC 2017] Sabha
 He published Shabda Kalpadruma, a 3. Surendranath Banerjee — Founder of
Sanskrit language dictionary. the Indian Association
 He also wrote articles that were Which of the above pairs is/are correctly
published in Ishwar Chandra Gupta's matched?
newspaper Sambad Prabhakar. (a) 1 only
31. Gazulu Lakshminarasu Chetty [UPSC (b) 1 and 3 only
2017] (c) 2 and 3 only
 He was an Indian merchant, Indian (d) 1, 2 and 3
independence activist and political
activist who founded the Madras Native
Association.
 He was the founder of first south Indian
newspaper ―cresent‖
 He was also the second Indian to be
appointed a member of the Madras
Legislative Council.
32. Surendranath Banerjee/Rashtraguru
 Indian Association was the first declared
nationalist organization founded in
British India by Surendranath Banerjee
and Anand Mohan Bose in 1876. [UPSC
2017]

Content made from Every word of Previous Year Paper | www.sunyaias.com | 8279688595
Page. 16
PRELIMS PYQs Content- (Modern History)
 He founded The Bengalee, a newspaper.
 He welcomed Montagu–Chelmsford
Reforms, unlike Congress, and with
many liberal leaders he left Congress and
founded new organisation named Indian
National Liberation Federation in 1919.
 He was also known as Indian Edmund
Burke.
33. V.O Chidambaram Pillai 1. Who of the following organized a march
 Was popularly known as ‗Kappalottiya on the Tanjore coast to break the Salt Law
Tamizhan‘ or ―The Tamil Helmsman‖ in April 1930?
 Swadeshi Steam Navigation Company (a) V. O. Chidambaram Pillai
was founded by him in 1906 to challenge (b) C. Rajagopalachari
the monopoly of the British in sea trade. (c) K. Kamaraj
 He launched the first indigenous Indian (d) Annie Besant
shipping service between Tuticorin and
Colombo with the Swadeshi Steam
Navigation Company (SSNC),
competing against British ships.
 Writings: Meyyaram (1914), Meyyarivu
(1915), Anthology (1915), Thirukural
with literary notes of Manakudavar
(1917), Tholkappiam with literary notes
of Ilampooranar (1928), Autobiography
(1946).
34. C Rajagopalachari
 C. Rajagopalachari led Salt Disobedience
march from Tiruchirapally to
Vedaranniyam (off the Tanjore coast).
[UPSC 2015]
 He participated in Non – Cooperation
movement, agitations against Rowlatt
act, Civil disobedience movement and
Vaikom Satyagraha.
 He led the group of No Changers i.e.
individuals against contesting elections
for the Imperial Legislative Council.
 He wrote a Tamil translation of the
Ramayana, which was later published as
Chakravarthi Thirumagan.
 C R Formula was a proposal formulated by
Rajaji to end the deadlock between Indian
National Congress and All India Muslim
League.
35. K Kamraj
 Kumaraswami Kamaraj was an Indian
independence activist and politician
widely acknowledged as "Kingmaker" in

Content made from Every word of Previous Year Paper | www.sunyaias.com | 8279688595
Page. 17
PRELIMS PYQs Content- (Modern History)
Indian politics during the 1960s.
 Kamraj Plan: In 1963 he suggested to
Nehru that senior Congress leaders
should resign from ministerial posts to
take up organisational work -> designed
primarily to dispel from the minds of
Congressmen the lure of power.
36. Annie Besant: She founded the All India Home
Rule League in 1916. This was India's first
faction to demand complete independence +
She was one of the founders of the Banaras
Hindu University + She also founded the
Central Hindu School to promote education +
By 1918, she had founded Madras Parliament,
Madanapalle College (now in Andhra Pradesh),
Adyar Arts League, Bombay Home Rule
League, Women's Indian Association at Adyar
and the Women's Indian Association at Adyar.
37. Hind Mazdoor Sabha 4. Who among the following were founders
 It is a National Trade Union Centre in of the ―Hind Mazdoor Sabha‖ established
India founded in Howrah in West Bengal in 1948?
by Socialist, Forward Bloc follower and (a) B. Krishna Pillai, K.C. George and
independent unionists. E.M.S. Namboodiripad
 Its founders included Basawan Singh (b) Jayaprakash Narayan, Deen Day al
(Sinha), Ashok Mehta, R.S. Ruikar, Mani Upadhyay and M.N. Roy
Benkara, ShibnathBenerajee, R.K. (c) C.P. Ramaswamy Iyer, K. Kamaraj
Khedgikar, T.S. Ramanujam, VS. and Veeresalingam Pantulu
Mathur, G.G. Mehta. [UPSC 2018] (d) Ashok Mehta, T.S. Ramanujam and
 R.S. Ruikar was elected president and G.G. Mehta
Ashok Mehta as its General Secretary.
 It absorbed the Royists Indian Federation
of Labour and the Socialist Hind
Mazdoor Panchayat.
 It was intended to be a third force in
Indian trade unionism, balancing
between INTUC on one side and AITUC
on the other.
38. Krishna Pillai
 Described as ―Kerala‘s First Communist‖
 He was the founder secretary of the
Communist party in Kerala in 1937 and
was pioneer in working class movement.
 During Civil Disobedience movement, he
defended the national flag and resisted
lathi charge on the Calicut beach on
November 11, 1930. He later founded the
Kerala Communist Movement.
39. EMS Namboodiripad
 He was born on 13 June 1909 into an
Content made from Every word of Previous Year Paper | www.sunyaias.com | 8279688595
Page. 18
PRELIMS PYQs Content- (Modern History)
aristocratic upper-caste Brahmin family
in the Elamkulam village in Kerala.
 He mobilised campaigns aimed at
normalising inter-caste marriage and widow
remarriage in the Namboothiri community
in Kerala.
 He was imprisoned for 1 year for
participation in Civil Disobedience
Movement.
 He co-founded a socialist wing within
the congress called the Congress
Socialist Party (CSP).
 He was also famously known to refer to
Gandhi as ―Hindu fundamentalist‖.
 In 1939, he officially joined CPI after
disillusioned with Congress‘ diplomatic
compromise with the British government
during the Second World War.
40. KC George: He was an Indian communist
leader from Alleppey, Kerala and one of the
founding leaders of the Communist Party of
India in Travancore. He was the Member of
Rajya Sabha during 1952 to 1954.
41. Jayaprakash Narayan
 He joined the Indian National Congress
in 1929 and later played role in formation
of Congress Socialist Party (1934), a left-
wing group within the Congress Party.
 He left the Congress Party in 1948 and
initiated an anti-Congress Campaign;
formed Praja Socialist Party in 1952.
 started the Bhoodan Yajna Movement of
Vinoba Bhave in 1954 which demanded
land redistribution to the landless.
 Fought for ―reconstruction of Indian
polity‖ -> four-tier hierarchy of village,
district, state, and union councils
(Chaukhamba Raj).
 He started a program for social
transformation named ‗Sampoorna
Kranti' (total revolution) in 1974 against
corruption in public life. It was to bring
in a change in the existing society that is
in tune with the ideals of the Sarvodaya
(Gandhian philosophy- progress for all).
 Rejected the ultimate solution of
―revolution‖ to bring down capitalism as
being advocated by the Marxists and

Content made from Every word of Previous Year Paper | www.sunyaias.com | 8279688595
Page. 19
PRELIMS PYQs Content- (Modern History)
instead advocated Socialism.
42. Deen Dayal Upadhyay
 Born in Mathura, UP in 1916, Deendayal
Upadhyay was one of the most important
leaders of the Bharatiya Jana Sangh, the
forerunner of the present day Bharatiya
Janata Party.
 He qualified civil services examination,
where he got his nickname as Panditji for
appearing in the examination hall
wearing traditional dhoti-kurta and cap.
 In 1942 he became a life-time volunteer
of Rashtriya Swayamsevak Sangh (RSS).
 He started Rashtra Dharma from
Lucknow in 1940s meant for spreading
ideology of Hindutva nationalism.
 He started the weekly ―Panchjanya‖ and
daily ―Swadesh‖.
 Other Important Literary works are
Samrat Chandragupta (1946), Jagatguru
Shankaracharya (1947), Akhand Bharat
Kyon? (1952), The Two Plans: Promises,
Performances, Prospects (1958), Rashtra
Jivan Ki Samasyayen (1960), Political
Diary (1968), Rashtra Chintan and
Rashtra Jivan Ki Disha.
43. MN Roy
 M. N. Roy was famous as the Father of
Indian communism and first
revolutionary leader of India.
 Under the influence of the Bolshevik
revolution, Roy along with the American
Leftists and Mexican unionists and
anarchists founded Mexican Communist
Party (PCM) in November 1917.
 In 1922, he prepared detailed programme
for the consideration of Indian National
Congress -> proposed nationalization of
railways, mines, water ways.
 He suggested that aim of the Congress
party should be complete national
independence from British domination.
 He established Communist Party of India
in Tashkent in 1925 and was also creator
of the Mexican Communist Party.
44. CP Ramaswamy Iyer
 He was the Dewan of Travancore
princely state from 1936 to 1947.

Content made from Every word of Previous Year Paper | www.sunyaias.com | 8279688595
Page. 20
PRELIMS PYQs Content- (Modern History)
 He was first person in India to suggest a
plan for interlinking rivers in the country.
 He established the Pallivasal Hydro-
electric power project on the Periyar
River and initiated the Pechipara Hydro-
electric scheme and the Periyar Wildlife
Sanctuary Project.
 Travancore became the first state to
nationalize road transport in India. The
first cement highway in India between
Trivandrum and Kanyakumari covering a
distance of 88 Kms was constructed
during his tenure.
 He started the University of Travancore
(present University of Kerala) in 1937.
 He was the first to introduce the mid-day
meal scheme in the form of the Vanchi
Poor Fund in Travancore to encourage
poor children to attend school.
 He established the Travancore Bank
which became State Bank of Travancore.
45. K Kamraj
 Kumaraswami Kamaraj was an Indian
independence activist and politician
widely acknowledged as "Kingmaker" in
Indian politics during the 1960s.
 Kamraj Plan: In 1963 he suggested to
Nehru that senior Congress leaders
should resign from ministerial posts to
take up organisational work -> designed
primarily to dispel from the minds of
Congressmen the lure of power.
46. Veeresalingam Panthulu
 He is considered as the Father of
renaissance movement in Telugu.
 He encouraged women education,
remarriage of widows and fought against
dowry system.
 He started a school in Dowlaiswaram in
1874 and ‗Hithakarini School‘ in 1908 in
Andhra Pradesh.
 His novel Rajasekhara Charitramu is
considered to be the first novel in Telugu
 He is the first person to conduct a widow
remarriage in Andhra Pradesh, the first to
start a co-educational school.

*******
Content made from Every word of Previous Year Paper | www.sunyaias.com | 8279688595
Page. 21
PRELIMS PYQs Content- (Modern History)
CHAPTER 3: CONSTITUTIONAL REFORMS DURING BRITISH INDIA
PRELIMS PYQ CONTENT – UPSC CSE EXAM – Topic-wise Segregated

1. Regulating Act of 1773 7. By which one of the following Acts was


 Passed by the British Parliament after the Governor General of Bengal
Lord North presented his bill in designated as the Governor General of
Parliament on May 18, 1773. India?
 First step taken by the British (a) The Regulating Act
Government to control and regulate (b) The Pitt's India Act
affairs of EIC in India. (c) The Charter Act of 1793
 Recognized for first time, political and (d) The Charter Act of 1833
administrative functions of Company;
 It laid the foundations of central
administration in India.
 Designated ‗Governor of Bengal' as the
Governor-General of Bengal.
 Created an Executive Council of four
members to assist him; first Governor
General was Lord Warren Hastings.
 Governors of Bombay and Madras were
made subordinate to governor-general of
Bengal
 Establishment of a Supreme Court at
Calcutta (1774)- Sir Elijah Impey served
as the Chief Justice, while Lemester,
Chambers, and Hyde were the other
judges of the court established in 1774.
 Banned Company officials from
engaging in any private trade or
accepting bribes from natives.
2. Pitt’s India Act of 1784
 It separated the East India Company's
political and corporate activities.
 From now Court of Directors to manage
the commercial affairs; Board of Control
to manage the political affairs (system of
Double Government).
 Company‘s territory was called British
Possessions.
 This Board of Control's members are:
The Chancellor of the Exchequer, The
Secretary of State, and Four Privy
Councilors are among the six officials
appointed by the King
 The Secretary of State was appointed as
the Board of Control's President.
 Governor-General was granted veto
power.

Content made from Every word of Previous Year Paper | www.sunyaias.com | 8279688595
Page. 22
PRELIMS PYQs Content- (Modern History)
 GG of Bengal cannot declare war without
permission.
 Provisions for Property disclosure:
Within two months of taking up their
posts, all civil and military officers of
EIC were ordered to provide the Court of
Directors with a full inventory of their
property in India and in Britain.
3. Charter Act of 1793
 The company's dominance over the
British possessions in India was
maintained by this Act. The company's
trade monopoly in India was extended for
another 20 years.
 The Act declared that "gain of
sovereignty by Crown subjects is on
behalf of the Crown and not in its own
right," implying that the company's
political activities were carried out on
behalf of the British government.
 The Board of Control's membership
changed. It was to consist of a President
and two junior members who were not
necessarily Privy Council members.
 This Act separated the company's
revenue administration and judicial
functions, resulting in the demise of Maal
Adalats (revenue courts).
4. Charter Act of 1833
 Governor-General of Bengal designated
as the Governor-General of India vested
with civil and military powers ->
Centralization of administration; Lord
William Bentick was the first Governor-
General of India.
 Deprived governors of Bombay and
Madras of their legislative powers.
 Ended commercial activities of the EIC
and was now purely administrative body.
 Attempted to introduce a system of open
competition for selection of civil servants
without any discrimination Indians.
 Governor General in Council to set up
Indian law commission.
 The Governor-General in council could
repeal, amend and alter any laws or
regulations enforced in British India.
 Decision of the Governor-General will

Content made from Every word of Previous Year Paper | www.sunyaias.com | 8279688595
Page. 23
PRELIMS PYQs Content- (Modern History)
prevail at the time of dispute on any
matter in the Governor-General counsels
and he can reject the majority decision
 All the commercial Privileges of the
company were abolished.
 Europeans and British could now freely
acquire, hold or dispose of any property
in India.
 The president of the Board of Control
was replaced by Minister for Indian
affairs
 Provided for Creation of the presidency
of Agra and Fort William by the division
of the Bengal presidency but this
provision never came into existence.
 First act that made provision to freely
admit the natives of India to share an
administration in the country.
 The act also provided for the diminution
of slavery existing in British India->
Slavery was abolished in India with the
act Act V of 1843.
 Increased the number of Bishopries to
three and made the Bishop of Calcutta the
Metropolitan Bishop of India. [UPSC
2023]
5. Charter Act, 1813 4. Consider the following statements about
 Given the East India Company a ‗the Charter Act of 1813‘:
monopoly to trade with East for a period 1. It ended the trade monopoly of the
of 20 years. East India Company in India except
 Effect of continental system of Europe for trade in tea and trade with China.
and Sanctions by Napoleon on British 2. It asserted the sovereignty of the
goods creating pressure on trade of East British Crown over the Indian
India Company as British Traders territories held by the Company.
demanded entry to the ports of Asia and 3. The revenues of India were now
dissolve the monopoly of the East India controlled by the British Parliament.
Company. Which of the statements given above are
 Ended the monopoly of East India correct?
Company except for tea and China (a) 1 and 2 only
 Empowered the Local Governments in (b) 2 and 3 only
India to impose taxes on persons and to (c) 1 and 3 only
punish those who did not pay them. (d) 1, 2 and 3
 Granted permission to the persons who
wished to go to India for promoting
moral and religious improvements.
 Company was asked to keep its territorial
and commercial accounts separate.
6. Charter Act of 1813 [UPSC 2019]

Content made from Every word of Previous Year Paper | www.sunyaias.com | 8279688595
Page. 24
PRELIMS PYQs Content- (Modern History)
7. Queen Victoria’s Proclamation (1858) 1. What was/were the object/objects of
[UPSC 2014] Queen Victoria‘s Proclamation (1858)?
8. GOVERNMENT OF INDIA ACT 1858 1. To disclaim any intention to annex
 Abolition of Company's rule and Indian States
beginning of the rule of British Crown 2. To place the Indian administration
 Abolition of Board of Control and Court under the British Crown
of Directors 3. To regulate East India Company‘s
 Created a new office Secretary of State trade with India
who would be a member of the British Select the correct answer using the code
Cabinet rule India with help of a Council given below.
 India Council consisting of 15 members (a) 1 and 2 only
 Sir Charles Wood last President of the (b) 2 only
Board of Control, was made the first (c) 1 and 3 only
secretary of State for India. (d) 1, 2 and 3
 Changed the designation of the
Governor-General of India to that of
Viceroy of India.
 Lord Canning the first viceroy of India.
 Established 15 member council of India
to assist the Secretary of State for India.
9. Government of India Act (1919) (Montagu- 2. The Government of India Act of 1919
Chelmsford reforms) [UPSC 2015] clearly defined
 Separating the central and provincial (a) the separation of power between the
subjects thus allowed to make laws on judiciary and the legislature
their respective list of subjects. (b) the jurisdiction of the central and
 The provincial subjects further divided provincial governments
into: - Transferred subjects - governor (c) the powers of the Secretary of State
has to deal with them on the advice of for India and the Viceroy
council of ministers responsible to (d) None of the above
legislative council + Reserved subjects -
governor has to deal with them on the 6. In the Government of India Act 1919, the
advice of council of ministers and here functions of Provincial Government
not responsible to legislative council were divided into ―Reserved‖ and
[System of Dyarchy]. ―Transferred‖ subjects. Which of the
 Justice Administration, Press, land following were treated as ―Reserved‖
Revenue, Forests, Labour Dispute subjects?
Settlements, Water, Agricultural Loans, 1. Administration of Justice
Police, and Prisons were among the items 2. Local Self-Government
on the Reserved List. [UPSC 2022] 3. Land Revenue
 The Secretary of State and the Governor- 4. Police
General had the authority to intervene in Select the correct answer using the code
things covered by the reserved list, but only given below:
to a limited extent in matters covered by the (a) 1, 2 and 3
transferred list. (b) 2, 3 and 4
 Bicameralism and direct elections in the (c) 1, 3 and 4
country. (d) 1, 2 and 4
 Communal representation was
consolidated as Sikhs, Indian Christians, 5. Consider the following statements:
1. The Montagu-Chelmsford Reforms

Content made from Every word of Previous Year Paper | www.sunyaias.com | 8279688595
Page. 25
PRELIMS PYQs Content- (Modern History)
Anglo-Indians and Europeans were given of 1919 recommended granting
separate electorate voting rights to all women above the
 Adult Suffrage was given on the basis of age of 21.
property, tax or education. 2. The Government of India Act of
 Establishment of a public service 1935 gave women reserved seats in
commission. the legislature.
 Appointment of a statutory commission Which of the statements given above
to inquire into and report-> Simon is/are correct?
commission was appointed (a) 1 only
 The legislators could now ask questions, (b) 2 only
pass adjournment motions and vote a part (c) Both 1 and 2
of the budget, but 75% of the budget was (d) Neither 1 nor 2
still not votable.
 communal representation was extended
to include Sikhs, Europeans and Anglo-
Indians.
 provision to provide reservation to the
non-Brahmins in Madras and the
depressed classes were also offered
nominated seats in the legislatures.
 It established an office of the High
Commissioner for India in London.
 ICS exam would be held in India-> 1st
ICS exam in Allahabad in 1920.
 Extension of Viceroy Executive
council: More Indians were added to
Viceroy‘s executive council by
Government of India Act 1919. For the
first time, an Indian was added in this
council by Government of India act
1909.
10. Government of India Act 1935
 It further extended franchise. About 10%
of the total population got the voting
right.
 The act abolished dyarchy in provinces
and introduced provincial autonomy in
its place. Consequently, the federal
subjects were divided into reserved and
transferred subjects. However, this
provision of the act did not come into
existence.
The act further extended principle of communal
representation by providing separate electorate
for depressed classes, women and labour.
[UPSC 2021]
11. Montagu-Chelmsford Reforms, 1919 3. Montague-Chelmsford Proposals were
[UPSC 2017] related to

Content made from Every word of Previous Year Paper | www.sunyaias.com | 8279688595
Page. 26
PRELIMS PYQs Content- (Modern History)
 The franchise was extended, and (a) social reforms
increased authority was given to central (b) educational reforms
and provincial legislative councils, but (c) reforms in police administration
the viceroy remained responsible only to (d) Constitutional reforms
London.
 In 1921, the "Dyarchy" was installed in
Bengal, Madras, Bombay, United
Provinces, M.P., Punjab, Bihar, Orissa
and Assam and in 1932, it was extended
to the North-West Frontier Province
 elected local councils were set up in rural
areas, and during the 1920s urban
municipal corporations were made more
democratic and "Indianized.
 number of Indians in Viceroy's Executive
Council would be three out of eight
members.
 act provided for the establishment of a
Public Service Commission in India for
the first time.
 was extended and Sikhs, Europeans and
Anglo Indians were included. The
Franchise (Right of voting) was granted
to the limited number of only those who
paid ‗ minimum tax‘ to the government
 Several members of the council including
Jinnah resigned in protest, passing of
Rowlattact to suppress opposition.

*******

Content made from Every word of Previous Year Paper | www.sunyaias.com | 8279688595
Page. 27
PRELIMS PYQs Content- (Modern History)
CHAPTER 4: BRITISH EXPANSION AND CONSOLIDATION IN INDIA
PRELIMS PYQ CONTENT – UPSC CSE EXAM – Topic-wise Segregated

1. Subsidiary Alliance [UPSC 2018] 1. Which one of the following statements


 It was an extension of Lord Wellesley‘s does not apply to the system of Subsidiary
policy of ring fence. Alliance introduced by Lord Wellesley?
 According to this system, the acceding (a) To maintain a large standing army at
ruler had to accept to pay a subsidy to the others expense
British for the maintenance of British (b) To keep India safe from Napoleonic
army. danger
 In return, British would protect them (c) To secure a fixed income for the
from their enemies which gave British Company
enormous expansion and promised non- (d) To establish British paramountcy over
interference in internal affairs of the the Indian States
Indian state.
 The allies of Indian state‘s ruler were
compelled to accept the permanent
garrison of British Army within their
territories.
 Indian state could not enter into any
alliance with any other foreign power
and could not employ any other
foreign nationals other than
Englishmen in his service.
 A British Resident was stationed in
the Indian Court.
 The term "Subsidiary Alliance" was
introduced by French Governor Dupleix.
2. States which Accepted Alliance
 The Indian princes who accepted the
subsidiary system were: the Nizam of
Hyderabad (September 1798 and 1800)
 the ruler of Mysore (1799)
 the ruler of Tanjore (October 1799)
 the Nawab of Awadh (November 1801)
 the Peshwa (December 1801)
 the Bhonsle Raja of Berar (December
1803)
 the Sindhia (February 1804)
 the Rajput states of iodhpur
 Jaipur, Macheri
 Bundi and the ruler of Bharatpur (1818).
The Holkars were the last Maratha
confederation to accept the Subsidiary
Alliance in 1818

*******

Content made from Every word of Previous Year Paper | www.sunyaias.com | 8279688595
Page. 28
PRELIMS PYQs Content- (Modern History)
CHAPTER 5: POPULAR UPRISINGS UPTO 1857
PRELIMS PYQ CONTENT – UPSC CSE EXAM – Topic-wise Segregated

1. Santhal Rebellion (1854) 1. After the Santhal Uprising subsided, what


 It was a revolt by the Santhal in was/were the measure/measures taken by the
present-day Jharkhand against the colonial government?
British East India Company and the 1. The territories called `Santhal Parganas‘
zamindari system. were created.
 The four Murmu Brothers - Sidhu, 2. It became illegal for a Santhal to transfer
Kanhu, Chand, and Bhairav - land to a non-Santhal.
spearheaded the revolt. Select the correct answer using the code
 Santhals of Rajmahal Hills resented given below:
the oppression by revenue officials, (a) 1 only
police, money-lenders, and (b) 2 only
landlords—in general, by the (c) Both 1 and 2
―outsiders‘ called dikus. (d) Neither 1 nor 2
 The Santhals under Sido and Kanhu
rose up against their oppressors,
declared the end of the Company‘s
rule and asserted themselves
independent in 1854.
 It was only in 1856 after extensive
military operations that the situation
was brought under control.
 A separate district of Santhal Parganas
was created by the Government to
pacify the Santhals. [UPSC 2018]
 British government enacted laws so
that the land of tribals could not be
taken by outsiders (dikus). [UPSC
2018]

*******

Content made from Every word of Previous Year Paper | www.sunyaias.com | 8279688595
Page. 29
PRELIMS PYQs Content- (Modern History)
CHAPTER 6: POPULAR UPRISINGS after 1857 (1857-1947)
PRELIMS PYQ CONTENT – UPSC CSE EXAM – Topic-wise Segregated

1. Indigo cultivation in India: Indigo 1. Indigo cultivation in India declined by the


cultivation started in Bengal in 1777 + In the beginning of the 20th century because of
nineteenth century, it was one of the most (a) peasant resistance to the oppressive
profitable commodities traded in the European conduct of planters
market often called ―Blue Gold‖ + There was (b) its unprofitability in world market
a significant increase in the Indigo production because of new inventions
in the 18th and 19th century in Bengal and (c) national leaders‘ opposition to the
Bihar due to increasing demand and cultivation of indigo
production + The cultivation of indigo fell (d) Government control over the planters
after the invention of Synthetic indigo, which
made natural indigo non-profitable for farmers
as well as traders + The quantity and value of
natural indigo were started declining in the
last decade of the 19th century and the
situation became worse in the 1st decade of
the 20th century. [UPSC 2020]
2. Indigo revolt (1859-60): It was led by
Digambar Biswas and Bishnu Biswas of
Nadia istrict Bengal + The local peasants were
forced by planters to grow indigo instead of
more profitable crops like rice; planters forced
peasants to take advanced sums and enter into
fraudulent contracts + If a farmer refused to
grow indigo and planted paddy instead, the
planters resorted to illegal means to get the
farmer to grow indigo such as looting and
burning crops, kidnapping the farmer‘s family
members, etc. + The revolt started from the
villages of Gobindapur and Chanugacha in
Krishnanagar, Gobindapur and Chaugacha in
Krishnanagar, Nadia district, where
Bishnucharan Biswas and Digambar Biswas
first led the rebellion against the planters in
Bengal, 1859. It spread rapidly in
Murshidabad, Birbhum, Burdwan, Pabna,
Khulna, and Narail + The revolt was
ruthlessly suppressed. Large forces of police
and military, backed by the British
Government and the zamindars, mercilessly
slaughtered a number of peasants + Finally,
the British government formed the Indigo
Commissionin 1860 due to Nawab Abdul
Latif's initiative with the goal of putting an
end to the repressions of indigo planters by
enacting the Indigo Act 1862.
3. Ulgulan or Great Tumult: During the last 2. With reference to the history of India,

Content made from Every word of Previous Year Paper | www.sunyaias.com | 8279688595
Page. 30
PRELIMS PYQs Content- (Modern History)
decade of the 19th century, the Mundas rose ―Ulgulan‖ or the Great Tumult is the
under Birsa Munda in a religious movement description of which of the following
(―ulgulan‖) with an agrarian and political events?
discontent + It aimed to establish a Munda Raj (a) The Revolt of 1857
in the region by killing thikadars and jagirdars (b) The Mappila Rebellion of 1921
+ The reason for the revolt is the British (c) The Indigo Revolt of 1859 – 60
replaced the traditional Khuntkari system of (d) Birsa Munda’s Revolt of 1899-1900
Munda tribals by the zamindari system in
1874. [UPSC 2020]
4. Birsa Munda: He is often referred to as
‗Dharti Abba‘ or the Earth Father + He led the
Ulgulan (revolt) or the Munda rebellion
against the British government-imposed
feudal state system + He organized masses to
stop paying debts to moneylenders and taxes
to the British + He combined religion with
politics and traveled across villages giving
discourses and building a politico-military
organization + His struggle led to the passage
of Chotanagpur Tenancy Act in 1908 which
restricted the passing of land from the tribal
people to non-tribals.

*******

Content made from Every word of Previous Year Paper | www.sunyaias.com | 8279688595
Page. 31
PRELIMS PYQs Content- (Modern History)
CHAPTER 7: SOCIO-RELIGIOUS REFORM MOVEMENTS
PRELIMS PYQ CONTENT – UPSC CSE EXAM – Topic-wise Segregated

1. All India Anti-Untouchability league: It 2. Consider the following pairs:


is a non-profit organization founded by Movement/ Leader
Mahatma Gandhi in 1932 to eradicate Organization
untouchability in India, working for Harijan 1. All India Anti- - League Mahatma
or Dalit people and upliftment of scheduled Untouchability Gandhi
castes of India. [UPSC 2019] 2. All India Kisan - Swami Sahajanand
2. All India Kisan Sabha: It was founded by Sabha Saraswati
Swami Sahajanand Saraswati in Lucknow 3. Self-Respect - E. V. Ramaswami
session of INC (1936) to spearhead peasant Movement Naicker
movements in colonial India. Sahajanand Which of the pairs given above is/are
was its first president and secretary was NG correctly matched?
Ranga. The motives of the All India Kisan (a) 1 only
Sabha were to abolish the Zamindari (b) 1 and 2 only
system, to reduce land revenue and to (c) 2 and 3 only
institutionalize credit. [UPSC 2019] (d) 1, 2 and 3
3. Self-Respect movement: It was founded in
1925 by S. Ramanathan who invited E. V.
Ramasamy (also called as Periyar by his
devoted followers) to head the movement
in Tamil Nadu, India against Brahminism.
It was an egalitarian movement that
advocated for the abolition of Brahminical
hegemony, equal rights for backward
classes and women in society, and the
revitalization of Dravidian languages such
as Telugu, Tamil, Kannada, and
Malayalam. [UPSC 2019]
4. Satya Shodhak Samaj 1. Satya Shodhak Samaj organized
 Jyotirao Phule along with his followers (a) a movement for upliftment of tribals in
formed Satyashodhak Samaj in 1873 Bihar
which meant ‗Seekers of Truth‘. (b) a temple-entry movement in Gujarat
 It seeks to attain equal social and (c) an anti-caste movement in
economic benefits for the lower castes in Maharashtra
Maharashtra. [UPSC 2016] (d) a peasant movement in Punjab
 He demanded representation of all
classes of the Hindus in all the local
bodies, in services and institutions and
also established a primary school for the
so-called untouchables in Poona.

*******

Content made from Every word of Previous Year Paper | www.sunyaias.com | 8279688595
Page. 32
PRELIMS PYQs Content- (Modern History)
CHAPTER 8: RISE OF MILITANT NATIONALISM
PRELIMS PYQ CONTENT – UPSC CSE EXAM – Topic-wise Segregated

1. Swadeshi Movement [UPSC 2019] 2. `Swadeshi‘ and ‗Boycott‘ were adopted as


 It had its genesis in opposition to the methods of struggle for first time during the
partition of Bengal in 1905. (a) agitation against Partition of Bengal
 It emphasised on self-reliance thus (b) Home Rule Movement
contributing to revival of indigenous (c) Non-Cooperation Movement
artisans craft and industries. (d) visit of the Simon Commission to India
 In August 1906 the National Council
of Education was established to 4. With reference to the Swadeshi Movement,
provide an impetus to program of consider the following statements:
national education as a part of 1. It contributed to the revival of the
Swadeshi movement. indigenous artisan crafts and industries.
 The Swadeshi and Boycott were adopted 2. The National Council of Education was
as methods of struggle for the first time established as a part of the Swadeshi
during the agitation against the Partition Movement.
of Bengal in Swadeshi movement. Which of given statements are correct?
[UPSC 2016] (a) 1 only
(b) 2 only
(c) Both 1 and 2
(d) Neither 1 nor 2
2. Surat Split, 1907 3. What was the main reason for the split in
 Surat Split refers to the splitting of the the Indian National Congress at Surat in
Congress party into 'Moderates' and 1907?
'Extremists' after a clash in 1907. (a) Introduction of communalism into
 The extremists were led by Lokmanya Indian politics by Lord Minto
Tilak, Lajpat Rai and Sri Aurobindo (b) Extremists’ lack of faith in the
and the moderates were led by Gopal capacity of the moderates to negotiate
Krishna Gokhale, Pherozeshah Mehta with the British Government
and Surendranath Banerjee. (c) Foundation of Muslim League
 Causes (d) Aurobindo Ghosh‘s inability to be
o Partition of Bengal in 1905 elected as the President of the Indian
provided impetus for the National Congress
Extremists to openly criticize the
Moderates' methods.
o The moderate method of
constitutional agitation had
exhausted the Indian masses,
giving rise to extremist tendencies
that emphasized direct action
against the British.
o The Extremists demanded that
Lala Lajpat Rai be appointed
President of the INC and
demanded for Swaraj's resolution-
> Both demands were rejected by
the Moderates.
3. The divided Congress re-united in the

Content made from Every word of Previous Year Paper | www.sunyaias.com | 8279688595
Page. 33
PRELIMS PYQs Content- (Modern History)
crucial Lucknow session of congress in
1916. [UPSC 2016]
4. Partition of Bengal (1905) 1. The Partition of Bengal made by Lord
 On October 16, 1905, the partition of Curzon in 1905 lasted until
Bengal province took effect during (a) the First World War when Indian troops
viceroyalty of Lord Curzon. were needed by the British and the
 The new province of East Bengal and partition was ended.
Assam, included the whole of Assam (b) King George V abrogated Curzon’s
and the Dacca, Rajshahi and Act at the Royal Darbar in Delhi in
Chittagong divisions with 1911
headquarters at Dacca. (c) Gandhiji launched his Civil
 Swadeshi or the use of Indian goods, and Disobedience Movement
the boycott of British goods were (d) the Partition of India, in 1947 when East
declared and pledged at mass meetings Bengal became East Pakistan
held throughout Bengal.
 Public burnings of foreign cloth were
organized in many places, and shops
selling foreign cloth were picketed.
 Atma Sakti (emphasis on self-reliance)
was an important aspect of the Swadeshi
Movement.
 Bengali students and women from
urban middle classes played an
important role in the Swadeshi
agitation.
 Many prominent Muslims such as
Abdul Rasul, a well-known barrister,
Liaquat Hussain, a well-known
agitator, and Guznavi, a businessman,
joined the Swadeshi Movement.
 King George V abrogated Curzon‘s
Act at the Royal Durbar in Delhi in
1911. Owing to mass political
protests, the partition of Bengal was
annulled in 1911. [UPSC 2014]

*******

Content made from Every word of Previous Year Paper | www.sunyaias.com | 8279688595
Page. 34
PRELIMS PYQs Content- (Modern History)
CHAPTER 9: FIRST PHASE OF REVOLUTIONARY ACTIVITIES (1907-17)
PRELIMS PYQ CONTENT – UPSC CSE EXAM – Topic-wise Segregated

1. Ghadr Party 1. The Ghadr (Ghadar) was a


 In November 1913, the Ghadar party was (a) revolutionary association of
founded by Lala Har Dayal with active Indians with headquarters at San
support from Ram Chandra, Barakatullah Francisco.
and Bhagwan Singh. (b) nationalist organization operating
 It was a revolutionary group organized from Singapore
around a weekly newspaper The Ghadr with (c) militant organization with
its headquarters at San Francisco and headquarters at Berlin
branches along the US coast and in the Far (d) communist movement for India‘s
East. [UPSC 2014] freedom with headquarters at
 Rash Bihari Bose, Raja Mahendra Pratap, Tashkent
Lala Hardayat, Abdul Rahim, Maulana
Obaidullah Sindhi, Champak Raman Pillai,
Sardar Singh Rana, and Madam Cama were
some of the prominent Indians Ghadar
Leaders who carried on revolutionary
activities and propaganda outside India.
[UPSC 2022]
 Ghadar movement was a secular democratic
movement whose tradition was later upheld
by Bhagat Singh with further addition of
socialist ideology.
 The party took its name Gadar to
consciously identify itself with first war of
Independence in 1857, which the British
termed the ―Gadar‖ (revolt).
2. Ghadr Party 2. Consider the following freedom
 In November 1913, the Ghadar party was fighters:
founded by Lala Har Dayal with active 1. Barindra Kumar Ghosh
support from Ram Chandra, Barakatullah 2. Jogesh Chandra Chatterjee
and Bhagwan Singh. 3. Rash Behari Bose
 It was a revolutionary group organized Who of the above was/were actively
around a weekly newspaper The Ghadr with associated with the Ghadar Party?
its headquarters at San Francisco and (a) 1 and 2
branches along the US coast and in Far East. (b) 2 only
 Rash Bihari Bose, Raja Mahendra Pratap, (c) 1 and 3
Lala Hardayat, Abdul Rahim, Maulana (d) 3 only
Obaidullah Sindhi, Champak Raman Pillai,
Sardar Singh Rana, and Madam Cama were
some of the prominent Indians Ghadar
Leaders who carried on revolutionary
activities and propaganda outside India.
[UPSC 2022]

*******
Content made from Every word of Previous Year Paper | www.sunyaias.com | 8279688595
Page. 35
PRELIMS PYQs Content- (Modern History)
CHAPTER 10: MAHATMA GANDHI: EMERGENCE AND IDEOLOGY
PRELIMS PYQ CONTENT – UPSC CSE EXAM – Topic-wise Segregated

1. Champaran Satyagraha (1917) 2. Which one of the following is a very


 It was the first Civil Disobedience significant aspect of the Champaran
movement in India. Satyagraha?
 The tenants from Champaran were forced (a) Active all-India participation of
under the law to plant three out of every lawyers, students and women in the
twenty parts of his land with indigo for National Movement
his landlord under Tinkathia system. (b) Active involvement of Dalit and
 In 1917, Gandhi visited Champaran Tribal communities of India in the
where Raj Kumar Shukla had drawn its National Movement
attention to exploitation of peasants by (c) Joining of peasant unrest to
European indigo planters. India’s National Movement
 Gandhi began his work from the house of (d) Drastic decrease in the cultivation of
Babu Gorakh Prasad in Motihari, plantation crops and commercial
headquarters of the then Champaran crops
district.
 His protest led to abolishing of
exploitative tinkathia system; victory at
Champaran established Gandhiji in
India‘s struggle against the British raj.
 It received support from nationalists such
as Rajendra Prasad and Anurag Narayan
Sinha.
 It was the first peasant movement to have
garnered nationwide attention. [UPSC
2018]
 Important Leaders: Brajkishore Prasad,
Rajendra Prasad, Anugrah Narayan Sinha
Ramnavmi Prasad, Mazhar-ul-Haq and
others including J. B. Kripalani & Babu
Gaya Prasad Singh.
2. Rowlatt Satyagraha [UPSC 2015] 1. With reference to Rowlatt Satyagraha,
 It was in response to the British which of the following statements is/ are
government enacting the Anarchical and correct?
Revolutionary Crimes Act of 1919, 1. The Rowlatt Act was based on the
popularly known as the Rowlatt Act. recommendations of the ‗Sedition
 The act was passed on the Committee‘.
recommendations of Sedition Committee 2. In Rowlatt Satyagraha, Gandhiji
chaired by Sir Sidney Rowlatt. tried to utilize the Home Rule
 It gave the government enormous powers League.
to repress political activities and allowed 3. Demonstrations against the arrival of
detention of political prisoners without the Simon Commission coincided
trial for two years. with Rowlatt Satyagraha.
3. While all sections of Indian political opinion Select the correct answer using the code
deeply resented the Rowlatt Act, it was left given below.
to Gandhi to suggest a practicable form of (a) 1 only
(b) 1 and 2 only

Content made from Every word of Previous Year Paper | www.sunyaias.com | 8279688595
Page. 36
PRELIMS PYQs Content- (Modern History)
all-India mass protest, going beyond (c) 2 and 3 only
petitioning but not intended to be (d) 1, 2 and 3
unrestrained or violent. The plan initially
was a rather modest one of volunteers
courting arrest by public sale of prohibited
works; it was extended by Gandhi on 23
March to include the novel and far more
radical idea of an all-India hartal on 30
March (later postponed to 6 April). The
brakes were there from the beginning,
however: the hartal was deliberately fixed
for a Sunday, and Gandhi explicitly declared
that ‗employees who are required to work
even on Sunday may only suspend work
after obtaining previous leave from their
employers‘. He also rejected the Arya
Samajist leader Swami Shrad- dhanand‘s
suggestion fora no-revenue call (‗Bhai
Sahcb! You will acknowledge that I am an
expert in Satyagraha business!'), and urged
the old Moderate leader Dinshaw Wacha to
accept his programme with the argument that
‗the growing generation will not be satisfied
with petitions, etc,... Satyagraha is the only
way, it seems to me, to stop terrorism‖ (letter
to Wacha, 25 February 1919).
In organizing his satyagraha, Gandhi tried to
utilize three types of political networks—the
Home Rule Leagues, certain Pan-Islamist
groups, and a Satyagraha Sabha which he
himself started at Bombay on 24 February.
Younger, radical members of the two Home
Rule Leagues were in need of a leader, as Besant
had suddenly turned into a moderate (she was
shouted down for supporting the Montford
reforms at the Delhi Congress of 1918), while
Tilak had left for England in September 1918.
Besant Home Rule

*******

Content made from Every word of Previous Year Paper | www.sunyaias.com | 8279688595
Page. 37
PRELIMS PYQs Content- (Modern History)
CHAPTER 11: NATIONALIST RESPONSE IN WAKE OF
SECOND WORLD WAR
PRELIMS PYQ CONTENT – UPSC CSE EXAM – Topic-wise Segregated

1. Cripps Mission [UPSC 2022, 2021, 2016]: 3. With reference to the proposals of Cripps
The proposal of the Cripps mission was that Mission, consider following statements:
―India would be a dominion associated with 1. The Constituent Assembly would
the United kingdom‖ + For the first time, have members nominated by the
British government recognized the ―Right of Provincial Assemblies as well as the
Dominion‘ for India + Indians were given Princely States.
promise of liberty to frame their own 2. Any Province, not prepared to accept
constitution + The Cripps mission which was the new Constitution would have the
a move to appease the Congress, Muslim right to sign a separate agreement with
League and Indian states at the same time was Britain regarding its future status.
rejected by all of them + Gandhi wanted an Which of given statements is/are correct?
undivided India, Muslim league wanted a (a) 1 only
separate Pakistan , Congress demanded a full (b) 2 only
control over defense ―stating that a slave (c) Both 1 and 2
country cannot have any inspiration‖ + Sikhs (d) Neither 1 nor 2
rejected because of non-accession of
provinces + Hindu Mahasabha rejected 4. With reference to 8th August 1942 in
because the ―Pakistan Virus‖ was alive + The Indian history, which one of the following
Dalits and depressed classed also rejected statements is correct?
because there was nothing new for them. (a) The Quit India Resolution was
2. Proposals of the Cripps Mission adopted by the AICC.
 An Indian Union with a dominion status (b) The Viceroy‘s Executive Council was
would be set up. It would free to expanded to include more Indians.
participate in the United Nations and other (c) The Congress ministries resigned in
international bodies. seven provinces.
 A Constituent Assembly would be formed (d) Cripps proposed an Indian Union with
to frame a new constitution for the full Dominion status once the Second
country. World War was over.
 The British Government would accept the
new constitution only on the following 2. The plan of Sir Stafford Cripps envisaged
conditions. that after the Second World War
o Any province not willing to join the (a) India should be granted complete
Union could have a separate independence
constitution and form a separate Union (b) India should be partitioned into two
o The transfer of power and the rights of before granting independence
minorities would be safeguarded by (c) India should be made a republic with
negotiations between the Constituent the condition that she will join the
Assembly and the British Government. Commonwealth
(d) India should be given Dominion
status

Content made from Every word of Previous Year Paper | www.sunyaias.com | 8279688595
Page. 38
PRELIMS PYQs Content- (Modern History)
3. Cabinet Mission (1946) [UPSC 2015] 1. With reference to cabinet mission, which
 Reached Delhi on March 24, and gave of the following statements is/are correct?
solution of constitutional problem in May 1. It recommended federal government.
 Objectives 2. It enlarged the powers of Indian
o To reach an agreement with Indian courts.
leaders on creation of a Constitution. 3. It provided for more Indians in the
o Creating a constitution-making body. ICS
o To form Executive Council with Select the correct answer using the code
support of major political parties. given below.
 Proposals (a) 1 only
o Rejection of demand for a full-fledged (b) 2 and 3
Pakistan as it would include a large (c) 1 and 3
non-Muslim population 38% in North- (d) None
West and 48% in North-East; Deep-
rooted ties would be jeopardized if
Bengal and Punjab were partitioned.
o Grouping of existing provincial
assemblies into three sections:
 Section-A: Madras, Bombay,
Central Provinces, United
Provinces, Bihar and Orissa
(Hindu-majority provinces).
 Section-B: Punjab, North-West
Frontier Province and Sindh
(Muslim-majority provinces).
 Section-C: Bengal and Assam
(Muslim majority provinces).
o Three-tier executive and legislature at
provincial, section and union levels.
o Constituent assembly was to be elected
by provincial assemblies by
proportional representation (voting in
3 groups— General, Muslims, Sikhs).
o Common centre would control
defense, communication & external
affairs. Union would have powers to
raise finances to manage the subjects.
o All subjects other than Union Subjects
would be vested in the provinces.
o Provinces were to have full autonomy
and residual powers.
o Princely states were no longer to be
under paramountcy of the British
government. They would be free to
enter into arrangement with successor
governments or British government.
4. Its long-term plan was accepted by Muslim
League on June 6, and by Congress on June
24, 1946.

Content made from Every word of Previous Year Paper | www.sunyaias.com | 8279688595
Page. 39
PRELIMS PYQs Content- (Modern History)
CHAPTER 12: CIVIL DISOBEDIENCE MOVEMENT AND ROUND TABLE
CONFERENCES
PRELIMS PYQ CONTENT – UPSC CSE EXAM – Topic-wise Segregated

1. Gandhi-Irwin Pact/Delhi pact (1931): It 2. The Gandhi-Irwin Pact included which of


was concluded between Mahatma Gandhi the following?
and the Lord Irwin in1931 + It was signed 1. Invitation to Congress to participate in
before the start of the second round table the Round Table Conference
conference in London + It placed the 2. Withdrawal of Ordinances promulgated
Congress on an equal footing with the in connection with the Civil
government. Disobedience Movement
 Gandhi on behalf of INC agreed to: 3. Acceptance of Gandhiji‘s suggestion
INC agreed to take part in the Round for enquiry into police excesses
Table Conference and stop the civil 4. Release of only those prisoners who
disobedience movement. were not charged with violence
 Irwin agreed for: Immediate release of Select the correct answer using the code
all political prisoners not convicted of given below :
violence, removal of the salt tax, (a) 1 only
remission of all fines not yet collected, (b) 1, 2 and 4 only
right to make salt in coastal villages for (c) 3 only
personal consumption, return of all lands (d) 2, 3 and 4 only
not yet sold to third parties, lenient
treatment to those government servants
who had resigned; right to peaceful and
non-aggressive picketing and withdrawal
of emergency ordinances. [UPSC 2020]
 Demands not accepted by Lord Irwin:
The commutation of death sentence of
Bhagat Singh, Rajguru and Sukhdev to a
life sentence, demand for a public
enquiry of excesses committed by police
during the civil disobedience movement.
2. Dandi March [UPSC 2019] 1. With reference to the British colonial rule
 It was the first act under the campaign of in India, consider following statements:
civil disobedience (satyagraha) of 1. Mahatma Gandhi was instrumental in
Gandhi waged against British rule. abolition of of ‗indentured labour‘
 On 12 March 1930 -> launch of salt
th 2. In Chelmsford‘s ‗War Conference‘,
satyagraha from Sabarmati with 78 Mahatma Gandhi did not support the
followers to the coastal town of Dandi recruitment of Indians for World
on the Arabian Sea. War.
 Despite Gandhiji‘s arrest, it continued. 3. Consequent upon the breaking of Salt
 Sarojini Naidu along with 2,500 Law by Indian people, the Indian
marchers led salt satyagraha on the National Congress was declared
Dharasana Salt Works. illegal by the colonial rulers.
 As the Salt March ended in month of Which of given statements are correct?
April 1930, several Congress Working (a) 1 and 2 only
Committees and leaders were arrested (b) 1 and 3 only
and INC was declared illegal by the (c) 2 and 3 only
British Administration. (d) 1, 2 and 3

Content made from Every word of Previous Year Paper | www.sunyaias.com | 8279688595
Page. 40
PRELIMS PYQs Content- (Modern History)
CHAPTER 13: QUIT INDIA MOVEMENT
PRELIMS PYQ CONTENT – UPSC CSE EXAM – Topic-wise Segregated

1. 8th August 1942: In July 1942, the CWC 1. With reference to 8th August 1942 in
met at Wardha and resolved that it would Indian history, which one of the
authorise Gandhi to take charge of the following statements is correct?
non-violent mass movement. The (a) The Quit India Resolution was
resolution generally referred to as the adopted by the AICC.
‗Quit India‘ resolution. Proposed by (b) The Viceroy‘s Executive Council was
Jawaharlal Nehru and seconded by Sardar expanded to include more Indians.
Patel, it was to be approved by the All (c) The Congress ministries resigned in
India Congress Committee meeting in seven provinces.
Bombay in August. (d) Cripps proposed an Indian Union with
full Dominion status once the Second
World War was over.

*******

Content made from Every word of Previous Year Paper | www.sunyaias.com | 8279688595
Page. 41
PRELIMS PYQs Content- (Modern History)
CHAPTER 14: IMPORTANT CONGRESS SESSIONS
PRELIMS PYQ CONTENT – UPSC CSE EXAM – Topic-wise Segregated

1. Surat Split, 1907 3. What was the main reason for the split in
 Surat Split refers to the splitting of the the Indian National Congress at Surat in
Congress party into 'Moderates' and 1907?
'Extremists' after a clash in 1907. (a) Introduction of communalism into
 The extremists were led by Lokmanya Indian politics by Lord Minto
Tilak, Lajpat Rai and Sri Aurobindo and (b) Extremists‘ lack of faith in the capacity
the moderates were led by Gopal Krishna of the moderates to negotiate with the
Gokhale, Pherozeshah Mehta and British Government
Surendranath Banerjee. (c) Foundation of Muslim League
 Causes [UPSC 2016] (d) Aurobindo Ghosh‘s inability to be
o Partition of Bengal in 1905 provided elected as the President of the Indian
impetus for the Extremists to openly National Congress
criticize the Moderates' methods.
o The moderate method of constitutional
agitation had exhausted the Indian
masses, giving rise to extremist
tendencies that emphasized direct
action against the British.
o The Extremists demanded that Lala
Lajpat Rai be appointed President of
the INC and demanded for Swaraj's
resolution-> Both demands were
rejected by the Moderates.
 The divided Congress re-united in the
crucial Lucknow session of congress in
1916.
2. INC Presidents [UPSC 2015] 2. Consider the following statements
 Sarojini Naidu was first Indian woman 1. First woman President of the Indian
president while Annie Besant was first National Congress was Sarojini Naidu.
woman president. 2. First Muslim President of INC was
 First male President was W.C. Banerjee. Badruddin Tyabji.
 First Muslim Male President was Which of given statements is/are correct?
Badruddin Tyabji. (a) 1 only
(b) 2 only
(c) Both 1 and 2
(d) Neither 1 nor 2
3. Important Congress sessions 1. The 1929 Session of the Indian National
 1929 Congress session at Lahore Congress is of significance in the history
o President: Jawahar Lal Nehru of the Freedom Movement because the
o Passed the resolution on ‗Poorna (a) Attainment of Self-Government was
Swaraj.‘ [UPSC 2014] declared as the objective of the
o Civil Disobedience movement for Congress
complete independence to be (b) Attainment of Poorna Swaraj Was
launched. adopted as the goal of the Congress
o 26 January to be observed as (c) Non-Cooperation Movement launched
‗Independence Day‘. (d) Decision to participate in the Round

Content made from Every word of Previous Year Paper | www.sunyaias.com | 8279688595
Page. 42
PRELIMS PYQs Content- (Modern History)
 1920 (Special Session): Calcutta Table Conference in London was
o President: Lala Lajpat Rai taken
o Mahatma Gandhi moved the Non-
cooperation resolution
 1906: Calcutta
o President: Dadabhai Naoroji
o Adopted four resolutions on: Swaraj
(Self Government), Boycott
Movement, Swadeshi & National
Education.
 1931: Karachi
o President: Vallabhbhai Patel
o Resolutions on Fundamental Rights
and National Economic Programme
o Endorsement of Gandhi-Irwin pact
o Gandhi nominated to represent INC in
the Second Round Table Conference
to be held in London.

*******

Content made from Every word of Previous Year Paper | www.sunyaias.com | 8279688595
Page. 43
PRELIMS PYQs Content- (Modern History)
CHAPTER 15: ORGANIZATIONS
PRELIMS PYQ CONTENT – UPSC CSE EXAM – Topic-wise Segregated

1. Hindu Mahasabha [UPSC 2018] 3. In 1920, which of the following changed


 Hindu Mahasabha was founded in 1914 by its name to ―Swarajya Sabha‖?
Madan Mohan Malviya. (a) All India Home Rule League
 First All India Hindu Mahasabha (b) Hindu Mahasabha
Conference was held at Hardwar in 1915. (c) South Indian Liberal Federation
 It was directly linked to RSS founded in (d) The Servants of India Society
1915 at Nagpur by K.B Hegdewar.
 In 1921, it changed its name to Akhil
Bharat Hindu Mahasabha.
2. South Indian Liberal Federation [UPSC
2018]
 It was founded by around 30 prominent
non-Brahmin leaders including Dr. C.
Natesanar, Sir Pitti Theyagarayar, T.M.
Nair and Alamelu Mangai Thayarammal
in 1915.
 The association started publishing three
newspapers: Dravidian in Tamil, Justice in
English and Andhra Prakasika in Telugu,
to propagate the ideals of the Party.
 Main objectives are to promote the
educational, social, economic, political
progress of all the communities other than
the Brahmins of South India and to work
for Non-Brahmin development by
constitutional government.
3. Servants of India Society [UPSC 2018]
 Servants of India Society was founded in
1905 by Gopal Krishna Gokhale, with the
support of dedicated individuals like
Gopal Krishna Deodhar, Anant
Patwardhan, Natesh Appaji Dravid, MG
Ranade and so on.
 It was country's first secular organization
dedicated to the disadvantaged, rural and
tribal peoples, emergency relief work,
literacy, and other social issues.
 The organization has its headquarters in
Pune (Poona) and branches in Chennai
(Madras), Mumbai (Bombay), Allahabad,
Nagpur, and other locales.
4. Hind Mazdoor Sabha 4. Who among the following were the
 It is a National Trade Union Centre in founders of the ―Hind Mazdoor Sabha‖
India founded in Howrah in West Bengal established in 1948?
by Socialist, Forward Bloc follower and (a) B. Krishna Pillai, K.C. George and
E.M.S. Namboodiripad

Content made from Every word of Previous Year Paper | www.sunyaias.com | 8279688595
Page. 44
PRELIMS PYQs Content- (Modern History)
independent unionists. (b) Jayaprakash Narayan, Deen Day al
 Its founders included Basawan Singh Upadhyay and M.N. Roy
(Sinha), Ashok Mehta, R.S. Ruikar, Mani (c) C.P. Ramaswamy Iyer, K. Kamaraj and
Benkara, ShibnathBenerajee, R.K. Veeresalingam Pantulu
Khedgikar, T.S. Ramanujam, VS. Mathur, (d) Ashok Mehta, T.S. Ramanujam and
G.G. Mehta. [UPSC 2018] G.G. Mehta
 R.S. Ruikar was elected president and
Ashok Mehta as its General Secretary.
 It absorbed the Royists Indian Federation
of Labour and the Socialist Hind Mazdoor
Panchayat.
 It was intended to be a third force in
Indian trade unionism, balancing between
INTUC on one side and AITUC on the
other.
5. Congress Socialist Party [UPSC 2015] 1. With reference to Congress Socialist Party,
 It was a left-wing group within the consider the following statements:
Congress; formed with Acharya Narendra 1. It advocated the boycott of British
Deva as President and Jay Prakash goods and evasion of taxes
Narayan as General Secretary in 1934. 2. It wanted to establish the dictatorship
 By 1935, one third of the Congress of the proletariat.
members were Congress Socialists. 3. It advocated a separate electorate for
 These leaders rejected the idea of Gandhi minorities and oppressed classes.
(which they saw as anti-rational). Which of the statements given above is/are
 They were influenced by Marxism- correct?
Leninism. (a) 1 and 2 only
 The CSP advocated decentralized (b) 3 only
socialism in which co-operatives, trade (c) 1, 2 and 3
unions, independent farmers, and local (d) None
authorities would hold a substantial share
of the economic power.
 As secularists, they hoped to transcend
communal divisions through class
solidarity.
6. Keshub Chandra Sen 2. Consider the following:
 He was involved with the activities of the 1. Calcutta Unitarian Committee
British Indian Association in his early 2. Tabernacle of New Dispensation
life. 3. Indian Reform Association
 He joined the Brahmo Samaj in 1857 and Keshab Chandra Sen is associated with the
was considered to be youthful entity of the establishment of which of the above?
movement. (a) 1 and 3 only
 Sangat Sabha was established in 1860 to (b) 2 and 3 only
promote mutual spiritual intercourse -> (c) 3 only
sowed the seeds of new Brahmoism in (d) 1, 2 and 3
syncretism with tenets of Christianity so
as to form a truly universal religion.
 In 1862, he helped found the Albert
College and were also instrumental in the

Content made from Every word of Previous Year Paper | www.sunyaias.com | 8279688595
Page. 45
PRELIMS PYQs Content- (Modern History)
launching of the Bethune College for
ladies and a number of schools in general.
 He started ―Indian Reform Association‖
and was appointed secretary of the Asiatic
Society in 1854. [UPSC 2016]
 In 1868, Keshub laid the foundation stone
of his new church, the Tabernacle of New
Dispensation.
 Indian reform association was founded in
1870 with Keshub Chunder Sen as
president. It represented the secular side
of the Brahmo Samaj.
7. Calcutta Unitarian committee: It was
established in 1823 by Rammohun Roy,
Dwarkanath Tagore, and William Adam.

*******

Content made from Every word of Previous Year Paper | www.sunyaias.com | 8279688595
Page. 46
PRELIMS PYQs Content- (Modern History)
CHAPTER 16: EVOLUTION OF CIVIL SERVICES IN INDIA
PRELIMS PYQ CONTENT – UPSC CSE EXAM – Topic-wise Segregated

1. Fort William College (Calcutta): By 1800s 1. Wellesley established the Fort William
the British territories in India greatly expanded College at Calcutta because
and there was an urgent need for more (a) He was asked by the Board of
thoroughly organized administration + This Directors at London to do so
required trained men who could not only (b) He wanted to revive interest in
communicate in local languages but also oriental learning in India
understand the country they were gaining (c) He wanted to provide William
control + Indian men were supposed to learn Carey and his associates with
native languages and even paid a sum called employment
Munshi‘s allowance to pay their Indian (d) He wanted to train British civilians
language tutors. It was to train these British for administrative purpose in India
officials that Fort William college was
established by Lord Richard Wellesley in 1800
+ The court of directors of the British East
India Company was never in support of a
training college in Kolkata (Calcutta) + Later a
separate College, East Indian Company
College was established in 1807 at England.
[UPSC 2020]

*******

Content made from Every word of Previous Year Paper | www.sunyaias.com | 8279688595
Page. 47
PRELIMS PYQs Content- (Modern History)
CHAPTER 17: DEVELOPMENT OF EDUCATION
PRELIMS PYQ CONTENT – UPSC CSE EXAM – Topic-wise Segregated

1. Charter Act of 1813: It incorporated the 1. Which of the following led to the
principle of encouraging learned Indians and introduction of English Education in
promoting knowledge of modern sciences in India?
the country. The Act directed the company to 1. Charter Act of 1813
sanction one lakh rupees annually for this 2. General Committee of Public
purpose. [UPSC 2018] Instruction, 1823
2. In 1823, the Governor -General -in Council 3. Orientalist and Anglicist
appointed a “General Committee of Public Controversy
Instruction”, which had the responsibility to Select the correct answer using the code
grant the one lakh of rupees for education. given below:
[UPSC 2018] (a) 1 and 2 only
3. Lord Macaulay's Minute (1835) minute settled (b) 2 only
the row in favour of Anglicists—the limited (c) 1 and 3 only
government resources were to be devoted to (d) 1, 2 and 3
teaching of western sciences and literature
through the medium of English language alone.
[UPSC 2018]
4. Wood’s Despatch
 Grants-in-Aid system: It recommended a
system of grants-in-aid to encourage
private enterprise. [UPSC 2018]
 Establishment of Universities:
Universities on the model of the London
University be established in big cities
such as Bombay, Calcutta and Madras.
[UPSC 2018]
 Medium of instruction: It recommended
English as medium for instruction for
higher studies and vernacular‘s at school
level. It systemized hierarchy from
vernacular primary schools in villages at
bottom, followed by Anglo-vernacular
high schools and an affiliated college at
the district level.
 It laid stress on female and vocational
education and on teacher‘s training.
 Recommended for secular education in
government institutions.
 Asked GOI to assume responsibility for
education of masses.
5. Hunter Education commission (1882)
 It mostly confined its recommendations to
primary and secondary education.
 Recommended transfer of control of
primary education to district and
municipal boards

Content made from Every word of Previous Year Paper | www.sunyaias.com | 8279688595
Page. 48
PRELIMS PYQs Content- (Modern History)
 Secondary education should have two
divisions-> literary leading up to
university and vocational for commercial
careers.
6. Saddler University commission (1917)
 Separate board of secondary and
intermediate education.
 Less rigidity in framing University
regulations.
 Female education, applied scientific and
technological education, teacher‘s training
including for professional colleges should
be extended.
7. Hartog committee (1929)
 Emphasis to primary education; no hasty
expansion in education.
 Average students should be diverted o
vocational courses after 8th standard.
 Admissions in Universities should be
restricted to improve standards in the
University education.
8. Educational Institutions [UPSC 2018] 2. With reference to educational institutes
 Sanskrit College: In 1791, Jonathan during colonial rule in India, consider
Duncan started the Sanskrit College at the following pairs:
Benares. Institution Founder
 Calcutta Madrasa: Established by 1 Sanskrit College William Jones
Warren Hastings in 1781 for the study of at Benaras
Muslim law and related subjects. 2 Calcutta Madrasa Warren Hastings
 Fort William College was set up by Lord 3 Fort William Arthur Wellesley
Richard Wellesley in 1800 for training of College
civil servants of the Company in Which of the pairs given above is/are
languages and customs of Indians. correct?
(a) 1 and 2
(b) 2 only
(c) 1 and 3
(d) 3 only

*******

Content made from Every word of Previous Year Paper | www.sunyaias.com | 8279688595
Page. 49
PRELIMS PYQs Content- (Modern History)
CHAPTER 18: ECONOMIC IMPACT OF BRITISH RULE IN INDIA
PRELIMS PYQ CONTENT – UPSC CSE EXAM – Topic-wise Segregated

1. Impacts of Industrial revolution on India: The 4. Which of following statements correctly


discovery of steam power created threat to the explains the impact of the Industrial
Indian textile industry + Indian farmers forced to Revolution on India during the first half
produce cotton plantation so that it can fuel of the nineteenth century?
English factories + Farmers were forced to grow (a) Indian handicrafts were ruined.
cash crops in place of food crops (b) Machines were introduced in Indian
(commercialization of agriculture) + Ruining of textile industry in large numbers.
Indian handicrafts + Deindustrialization in India (c) Railway were laid in many parts of
+ Positive impacts: Introduction of assembly line the country.
and factories, electricity development (d) Heavy duties were imposed on the
contributed to faster and more efficient imports of British manufacturers.
production of goods and materials.[UPSC
2020]
2. Impacts of British Rule in India 3. Economically, one of the results of the
 Commercialization of agriculture: It is a British rule in India in the 19th century
phenomenon where agriculture is governed was the
by commercial consideration i.e. certain (a) increase in the export of Indian
specialised crops began to be grown not for handicrafts
consumption in village but for sale in (b) growth in the number of Indian
international market. British rule dictated owned factories
growth of commercial crops like tea and (c) commercialization of Indian
indigo to fulfil the needs of European agriculture
markets. [UPSC 2018] (d) rapid increase in the urban
 Decline in Indian Handicrafts: Colonial population
rule negatively affected Indian industries
and handicrafts.
 New urban centers sprawled up like
Bombay and Madras, which was at the
cost of decline of old urban centers such
as Surat and Masulipatnam.
 Deindustrialization: Industrial revolution
in England, led to pouring of British
goods in India at an unprecedented rate,
which ruined the Indian handicraft
industry and led to de-industrialization.
3. Ryotwari Settlement 2. Who among the following was/were
 The system was devised by Capt. Alexander associated with the introduction of the
Read and Thomas Munro at the end of the Ryotwari Settlement in India during the
18th century and introduced by the latter British rule?
when he was governor (1820–27) of Madras. 1. Lord Cornwallis
[UPSC 2017] 2. Alexander Read
 Characteristics 3. Thomas Munro
o Cultivator is the owner of the land and Select the correct answer using the code
they reserve all the ownership rights given below:
such they could sell, mortgage or give (a) 1 only
land to anyone as a reward. (b) 1 and 3 only

Content made from Every word of Previous Year Paper | www.sunyaias.com | 8279688595
Page. 50
PRELIMS PYQs Content- (Modern History)
o Taxes may vary with the type of land (c) 2 and 3 only
as 50% tax for dry land and 60% tax (d) 1, 2 and 3
for wet or irrigated land.
o There was no mediator for the
collection of tax. The owner of land
directly paid tax to the British
Government.
o The free or barren land was in charge
of the government and the government
can cultivate this land with its share.
o If the peasants are unable to pay the
tax the land can be confiscated by the
British Government.
o It was first introduced in Tamil Nadu;
later extended to Maharashtra, Berar,
East Punjab, Coorg and Assam.
4. Economic critics of Colonialism 1. Who of the following was/were economic
 Dadabhai Naoroji, R. C. Dutt, Ranade, critic/ critics of colonialism in India?
Gokhale, G. Subramania Iyer, were 1. Dadabhai Naoroji
among those who grounded Indian 2. G. Subramania Iyer
nationalism firmly on the foundation of 3. R. C. Dutt
anti- imperialism by highlighting Select the correct answer using the code
economic critique of colonialism. [UPSC given below.
2015] (a) 1 only
 Theory of Drain of Wealth: Economic (b) 1 and 2 only
nationalism's central subject was the (c) 2 and 3 only
"drain of wealth" theory. The constant (d) 1, 2 and 3
flow of India‘s wealth to England without
adequate economic, commercial or
material reward has been described by the
early nationalists and economists as the
‗drain of wealth‘. The theory was
systematically developed by Dadabhai
Naoroji initially through his book Poverty
and Un-British rule in India, which was
further studied and expanded by R.P Dutt,
M.G Ranade etc.
 R C Dutt, published the book ―Economic
History of India‖, in which he wrote about
the entire economic system of the nation
under Britishers, since the Battle of
Plassey (1757).

*******

Content made from Every word of Previous Year Paper | www.sunyaias.com | 8279688595
Page. 51
PRELIMS PYQs Content- (Modern History)
CHAPTER 19: MISCELLANEOUS
PRELIMS PYQ CONTENT – UPSC CSE EXAM – Topic-wise Segregated

1. Alexander Rea, AH Longhurst, Robert 9. With reference to Indian History,


Sewell, James Burgess and Walter Elliot: Alexander Rea, A. H. Longhurst, Robert
They have conducted systematic and careful Sewell, James Burgess and Walter Elliot
excavations at Bhattiprolu, Ghantasala, were associated with
Amaravati etc. and exposed relic caskets which (a) archaeological excavations
created much curiosity among Indian scholars (b) establishment of English Press in
about various aspects of these valuable Colonial India
reliquaries. [UPSC 2023] (c) establishment of Churches in
Princely States
(d) construction of railways in Colonial
India
2. National Handloom Day [UPSC 2023] 10. Consider the following statements:
 In 2015, the Government of India decided Statement-I: 7th August is declared as
to designate the 7th August every year, as the National Handloom Day.
the National Handloom Day. The first Statement-II: It was in 1905 that the
National Handloom Day was inaugurated Swadeshi Movement was launched on
on 7 August 2015 by Prime Minister the same day.
Narendra Modi in Chennai. On this day, we Which one of the following is correct in
honour our handloom-weaving community respect of the above statements?
and highlight the contribution of this sector (a) Both Statement-I and Statement-II
in the socio-economic development of our are correct and Statement-II is the
country. correct explanation for Statement-
 The Swadeshi Movement which was I.
launched on 7th August, 1905 had (b) Both Statement-I and Statement-II
encouraged indigenous industries and in are correct and Statement-II is not
particular handloom weavers. In 2015, the the correct explanation for
Government of India decided to designate Statement-I.
the 7th August every year, as the National (c) Statement-I is correct but Statement-
Handloom Day. II is incorrect.
(d) Statement-I is incorrect but
Statement-II is correct.
3. Rakhmabai case of 1884: Rukhmabai Raut was 8. In the context of Indian history, the
born on 22 November,1864 was one of the first Rakhmabai case of 1884 revolved
female practicing women doctor + She was an around
Indian physician and feminist best known for 1. Women‘s right to gain education
being one of the first practising women doctors in 2. Age of consent
colonial India as well as being involved in a 3. Restitution of conjugal rights
landmark legal case involving her marriage as a Select the correct answer using the code
child bride between 1884 and 1888 + In 1885, given below :
after 12 years of her marriage, Dadaji Bhikaji (her (a) 1 and 2 only
husband) filed a petition in the court and sought (b) 2 and 3 only
―restitution of conjugal rights‖ + Rakhmabai (c) 1 and 3 only
Defense Committee was formed by Behramji (d) 1, 2 and 3
Malabari and Pandita Ramabai in her defence.
The case continued for 4 years until Dadaji was
compensated in 1888, outside of court. The case

Content made from Every word of Previous Year Paper | www.sunyaias.com | 8279688595
Page. 52
PRELIMS PYQs Content- (Modern History)
was instrumental in the drafting of the Age of
Consent Act in 1891 + The legal and social
controversies provoked by the case revolved
round notions of colonial law, marriage and
conjugality, and the prospect of state intervention.
[UPSC 2020
4. Butler Committee/Indian States Committee 6. The object of the Butler Committee of
(1927) 1927 was to?
 It was a three -member committee headed (a) Define the jurisdiction of the Central
by Harcourt Butler appointed to investigate and Provincial Governments.
and clarify the relationship between the (b) Define the powers of the Secretary
paramount power and the Princes. It visited of State for India.
16 states and submitted its report in 1929 (c) Impose censorship on national press.
[UPSC 2017] (d) Improve the relationship between
 Recommendations the Government of India and the
o It advised that the Viceroy (instead of Indian States.
Governor-General in Council) should
represent the Crown in all dealings with
the States.
o Paramountcy must remain supreme.
o States should not be handed over to an
Indian government in British India,
responsible to an Indian Legislature.
But it could be done with the consent of
states.
o The viceroy was made the Crown‘s
agent in dealing with states.
5. Features of Indian Feudalism 2. With reference to Indian history, which of
 From the post-Maurya period and Gupta the following is/are the essential
times, India‘s political and administrative elements of the feudal system?
developments tended to feudalise the state 1. A very strong centralized political
apparatus. authority and a very weak provincial
 It gradually developed from the beginning or local political authority
of the land grants. 2. The emergence of administrative
 Earliest land grants belonging to the first structure based on control and
century BC were given to the Buddhist possession of the land
priests and Brahmanas. 3. Creation of the lord-vassal
 In the post-Guptas period even relationship between the feudal lord
administrative officials were granted land. and his overlord
 The landed beneficiaries were given both Select the correct answer using the code
powers of taxation and coercion-> given below.
disintegration of the central authority. (a) 1 and 2 only
 Earliest epigraphic record of a land grants (b) 2 and 3 only
in India is a Saatavahana inscription of the (c) 3 only
first century BC. Such a land grant (d) 1, 2 and 3
included the rights that:
o royal troops could not enter such land
granted
o government officials and district police

Content made from Every word of Previous Year Paper | www.sunyaias.com | 8279688595
Page. 53
PRELIMS PYQs Content- (Modern History)
was not supposed to disturb such lands.
 Vassalage expressed the relation of
personal dependence and loyalty between
the lord and his vassals. [UPSC 2015]
 Hierarchy of feudal lords and Hereditary
administrative positions.
 Decentralisation of Power-> Samantas
granted lands instead of salary and
proceeded to seize ownership while
continuing to refer to themselves as vassals
of their rulers.
 Imposition of improper taxes and rent
exploited the labour class.
 Fragmentation of social formation-> Castes
were split up into thousands of other castes
and sub-castes.
6. Radcliffe committee: Radcliffe line was 1. Radcliffe Committee was appointed to
named after Sir Cyril Radcliffe, the chairman (a) Solve the problems of minorities in
of the Border Commissions. Radcliffe divided India
India into West Pakistan, East Pakistan and (b) Give effect to Independence Bill
India. (c) Delimit boundaries of India and
Pakistan
(d) Enquire into riots in East Bengal
7. Land Reforms 7. With reference to land reforms in
 The Land reform constitutes critical independent India, which one of the
measures to improve socio-economic following statements is correct?
condition of agricultural tenants. (a) The ceiling laws were aimed at
 Four Components family holdings and not individual
o The Abolition of the Intermediaries holdings.
o Tenancy Reforms (b) The major aim of land reforms
o Fixing Ceilings on Landholdings was to provide agricultural land to
o Consolidation of Landholdings. all the landless.
 Major aim: To distribute surplus land (c) It resulted in the cultivation of cash
among landless peoples. [UPSC 2019] crops as a predominant form of
cultivation.
(d) Land reforms permitted no
exemptions to the ceiling limits.

Content made from Every word of Previous Year Paper | www.sunyaias.com | 8279688595
Page. 54
PRELIMS PYQs Content- (Modern History)

8. Ceilings on Landholdings [UPSC 2019]


 It refers to legal stipulation of maximum
size beyond which no individual farmer or
farm household could hold any land.
 Aim: To deter the concentration of land in
the hands of a few.
 They were aimed at both family holdings
and individual holdings.
 Ceiling limits varied from state to state.
 National guidelines were issued in 1972
with ceiling limits varying from region to
region, depending on the kind of land, its
productivity, and other such factors.
 Exemptions allowed for plantations of
crops like tea and coffee, Bhoodan Yagna
committees, registered cooperatives and
other bodies.
9. Factories Act, 1881 3. Consider following statements:
 The 1881 Act aimed to improve working 1. The Factories Act, 1881 was
conditions of labour. passed with a view to fix the
wages of industrial workers and to
Content made from Every word of Previous Year Paper | www.sunyaias.com | 8279688595
Page. 55
PRELIMS PYQs Content- (Modern History)
 It is applicable only to factories using allow the workers to form trade
mechanical powers, employing not less unions.
than 100 workers. 2. N.M. Lokhande was a pioneer in
 It prohibited the employment of children organizing the labour movement
under the age of seven and required that in British India.
dangerous machinery should be fenced Which of the above statements is/are
properly. correct?
 Children between the age 7-12 were to (a) 1 only
work for maximum 9 hours. (b) 2 only
 Provision for one-hour rest during the (c) Both 1 and 2
working period and four days leave in a (d) Neither 1 nor 2
month for the workers.
 Four days leave in a month for workers was
made compulsory.
 There is no provision for fixed wage and
formation of trade unions.
10. NM Lokhande
 Narayan Meghaji Lokhande born in 1848
in Thane, Maharashtra, is the father of the
trade union movement in India.
 He was greatly influenced by Mahatma
Jyotirao Phule and his followers.
 In 1875, he was the first to represent
grievances of Indian working class before
labour commission of Bombay.
 He started publication of Deenbandhu
newspaper in Bombay.
 He also set up the first association of Indian
workers ―Bombay Mill-Hands association‖
in Bombay.
 He is remembered not only for
ameliorating the working conditions of
textile mill-hands in the 19th century but
also for his courageous initiatives on caste
and communal issues. [UPSC 2017]
11. Events: 4. With reference to Indian freedom
 Second Round Conference happened in struggle, consider the following events :
1931, Quit India Movement in 1942 and 1. Mutiny in Royal Indian Navy
Mutiny in Royal Indian Navy in 1946. 2. Quit India Movement launched
[UPSC 2017] 3. Second Round Table Conference
What is the correct chronological
sequence of the above events ?
(a) 1-2-3
(b) 2-1-3
(c) 3-2-1
(d) 3-1-2
12. Gopal Babu Walangkar: Also known as 5. The Vital Vidhvansak, the first monthly
Gopal Krishna, he was born into a family of journal to have the untouchable people

Content made from Every word of Previous Year Paper | www.sunyaias.com | 8279688595
Page. 56
PRELIMS PYQs Content- (Modern History)
the untouchable Mahar caste in Raigad district, as its target audience was published by
Maharashtra + He was the first to fight for the (a) Gopal Babu Walangkar
rights of the Mahars in Maharashtra + He (b) Jyotiba Phule
developed a racial theory to explain the (c) Mohandas Karamchand Gandhi
oppression and also published the first journal (d) Bhimarao Ramji Ambedkar
targeted at the untouchable people + He
claimed that "high-caste people from the south
were 'Australian–Semitic non-Aryans' and
African negroes, that Chitpavan Brahmans
were 'Barbary Jews', and that the high-caste
Marathas' forebears were 'Turks'" + He
published monthly journal titled Vital-
Vidhvansak (Destroyer of Brahmanical or
Ceremonial Pollution), which was the first to
have the untouchable people as its target
audience + He also wrote articles for Marathi-
language newspapers such as Sudharak and
Deenbandhu.
13. Jyotiba Phule: He was born in 1827 in
present-day Maharashtra and belonged to the
Mali caste of gardeners + He was influenced
by Thomas Paine‘s book titled The Rights of
Man and believed that the only solution to
combat the social evils was the enlightenment
of women and members of the lower castes +
Phule and his followers formed Satyashodhak
Samaj (Seekers of Truth) in 1873 to attain
equal social and economic benefits for the
lower castes in Maharashtra + He was
bestowed with the title of Mahatma by social
activist Vithalrao Krishnaji Vandekar + He
attacked the orthodox Brahmins and other
upper castes and termed them as "hypocrites" +
Important publications are Tritiya Ratna
(1855), Powada: Chatrapati Shivajiraje Bhosle
Yancha (1869), Gulamgiri (1873),
Shetkarayacha Aasud (1881).
14. Savitribai Phule: She was illiterate when she
married to Jyotiba Phule at the age of nine
years + Later Jyotiba admitted Savitribai to a
teachers‘ training Institute in Pune; and then
she became the first female teacher in India
(1848) + She also started the practice of
Satyashodhak Marriage, where couples took an
oath of education and equality + They also dug
a well at their courtyard for untouchables, who
had no access to public drinking water
facilities.
15. Mahatma Gandhi: Gandhiji was a prolific

Content made from Every word of Previous Year Paper | www.sunyaias.com | 8279688595
Page. 57
PRELIMS PYQs Content- (Modern History)
writer and he has written many articles
throughout his life. He edited several
newspapers including Harijan in Gujarati,
Indian opinion in South Africa, and Young
India in English + He wrote several books
including his autobiography ―The Story of My
Experiments with Truth‖ and Hind Swaraj +
He was associated with ―Songs from Prison‖, a
translation of ancient Indian religious lyrics in
English in 1934.
16. BR Ambedkar: He set up 'Bahishkrit Hitkarini
Sabha in 1923 to spread education and culture
amongst downtrodden + In 1936, he
founded Independent Labour Party + Important
works of Dr. Ambedkar are Mook Nayak
(weekly) 1920; Janta (weekly) 1930;
Annihilation of Caste 1936; The Untouchables
1948; Buddha or Karl Marx 1956, etc. [UPSC
2020]

*******

Content made from Every word of Previous Year Paper | www.sunyaias.com | 8279688595
Page. 58

You might also like